11.07.2015 Views

Self Assessment Program - American Diabetes Association

Self Assessment Program - American Diabetes Association

Self Assessment Program - American Diabetes Association

SHOW MORE
SHOW LESS
  • No tags were found...

Create successful ePaper yourself

Turn your PDF publications into a flip-book with our unique Google optimized e-Paper software.

®<strong>American</strong><strong>Diabetes</strong><strong>Association</strong>®Cure Care Commitment ®ADA-SAP Module 1Basic Principles of Management of Type 2 <strong>Diabetes</strong><strong>American</strong> <strong>Diabetes</strong> <strong>Association</strong><strong>Self</strong>-<strong>Assessment</strong> <strong>Program</strong>Multiple Choice Questions <strong>Assessment</strong> and Educational (Learning) Critiques Components© 2007 <strong>American</strong> <strong>Diabetes</strong> <strong>Association</strong>. All rights reserved.


PEIDeveloped and Published by: Professional Evaluation, Inc.Developing Medical Specialty Board Category 1 CME <strong>Program</strong>s for Over Three DecadesProfessional Evaluation Inc.4 Midland Avenue, Suite 105, Berwyn, PA 19312-1687 | www.proevalinc.com


<strong>American</strong> <strong>Diabetes</strong> <strong>Association</strong>TM<strong>Self</strong>-<strong>Assessment</strong> <strong>Program</strong> (ADA-SAP ) Module 1Basic Principles of Management of Type 2 <strong>Diabetes</strong>Table of Contents3 Learning Objectives3 Accreditation4 Faculty4 Disclosure Statement5 General Instructions6 Multiple Choice Questions (MCQs)25 Appendix26 I. Diagnosis of impaired glucose tolerance (IGT) and impaired fasting glucose (IFG)27 II. Diagnosis of type 2 diabetes28 III. Summary of recommendations for adults with diabetes34 Educational Critiques65 Computer-scored Answer Sheet66 <strong>Program</strong> Evaluation Questions


<strong>American</strong><strong>Diabetes</strong>® <strong>Association</strong>®C ure Care Commitment ®Dear Health Care Professional:The <strong>American</strong> <strong>Diabetes</strong> <strong>Association</strong> (ADA), in cooperation with Professional Evaluation, Inc. (PEI),has developed the <strong>American</strong> <strong>Diabetes</strong> <strong>Association</strong> <strong>Self</strong>-<strong>Assessment</strong> <strong>Program</strong> (ADA-SAP TM ) series.Module 1 of ADA-SAP TM series covers the Basic Principles of Diagnosis and Management of Type 2<strong>Diabetes</strong> in adults. Future modules in the ADA-SAP TM series will cover topics such as, combinationtherapy for type 2 diabetes, insulin use, and management of co-morbidities. For additional information,visit PEI’s website, www.proevalinc.com.The multiple choice questions component of Module 1 of ADA-SAP TM was developed by PEI at theupper level of difficulty similar in type and format to a board certifying examination. It was designedto objectively assess, strengthen and reinforce your knowledge and to provide you with an in-depthlearning experience of active engagement. Once you have properly completed Module 1 in it’s entirety(see General Instructions on page 5), your knowledge of the approach to diagnosis and managementof type 2 diabetes in adults should be greatly enhanced.The ADA looks forward to your participation in this important new program of continuing education,as part of your commitment to lifelong learning.Sincerely,Michael H. Davidson M.D., FACC, FACP<strong>Program</strong> ChairChief Medical Officer, PEIClinical Professor of Medicine, Director of Preventive CardiologyThe University of Chicago Pritzker School of Medicine2<strong>American</strong> <strong>Diabetes</strong> <strong>Association</strong> <strong>Self</strong>-<strong>Assessment</strong> <strong>Program</strong>


Target AudiencePrimary Care Physicians, General Internists, FamilyPhysicians, Doctors of Osteopathy, Registered Nurses,Nurse Practitioners, Pharmacists, Dietitians, andCertified <strong>Diabetes</strong> Educators.Learning ObjectivesBy completing this CME/CE enduring materialsactivity (Module 1), participants will be better able to: Discuss the pathogenesis of insulin resistanceand pancreatic β-cell dysfunction leading to type 2diabetes. Interpret and utilize ADA Guidelines andtreatment algorithms. Implement non-pharmacological and pharmacologicaltreatment strategies for hyperglycemiain type 2 diabetes.This enduring materials activity was originallyreleased on November 15, 2007 and terminatedon November 30, 2009.Faculty ChairMichael H. Davidson, M.D., FACC, FACPClinical ProfessorDirector of Preventive CardiologyThe University of ChicagoExecutive Medical DirectorRadiant ResearchChicago, IL FacultyElliot B. Davidson, M.D., FAAFPFamily MedicineAkron General Medical CenterAkron, OHStanley Schwartz, M.D., FACPClinical Associate Professor of MedicineDirector, <strong>Diabetes</strong> Disease ManagementUniversity of Pennsylvania Health SystemPhiladelphia, PA Chief Editor and PublisherDante S. LaRocca, Ph.D.PresidentProfessional Evaluation, Inc.Ambler, PA ReviewersM. Sue Kirkman, M.D.Vice President, Clinical Affairs<strong>American</strong> <strong>Diabetes</strong> <strong>Association</strong>Alexandria, VABelinda P. Childs,ARNP, MN, CDE, BC-ADMClinic and Research CoordinatorMid-America <strong>Diabetes</strong> Associates, PAWichita, KSRussell D. White,M.D., FAFP, FACSM, MACEProfessor of MedicineDirector,Sports Medicine Fellowship <strong>Program</strong>University of MissouriKansas City School of MedicineTruman Medical Center—LakewoodKansas City, MOThis activity was supported by an unrestricted educational grant from Merck & Co., Inc.<strong>American</strong> <strong>Diabetes</strong> <strong>Association</strong> <strong>Self</strong>-<strong>Assessment</strong> <strong>Program</strong>3


Disclosure StatementAll participating faculty, course directors, and planning committee members are required to disclose to the program audienceany financial relationships related to the subject matter of this program. Disclosure information is reviewed in advance inorder to manage and resolve any possible conflicts of interest. The intent of this disclosure is not to prevent a planner orpresenter from being involved in the activity, but rather to provide participants with information on which they can maketheir own judgments.Content Validation StatementThe <strong>American</strong> <strong>Diabetes</strong> <strong>Association</strong>’s accredited continuing education activities are based on evidence that is acceptedwithin the profession of medicine. Scientific research referred to, reported, or used in support or justification of a patientcare recommendation conforms to the generally accepted standards of experimental design, data collection and analysis.Unapproved Uses of Drugs/DevicesIn accordance with requirements of the FDA, the audience is advised that information presented in this continuingeducation activity may contain references to unlabeled or unapproved uses of drugs or devices. Please refer to the FDAapproved package insert for each drug/device for full prescribing/utilization information.Belinda P. Childs, ARNP, MN, CDE, BC-ADMResearch Support: Amylin, Bayer Healthcare, Biodel, Eli Lilly, Novartis, Roche Diagnostic, sanofi-aventis;Speakers Bureau: Amylin, Eli Lilly; Advisory Panel: Amylin; Consultant: Roche Diagnostic, sanofi-aventis.Elliot B. Davidson, M.D., FAAFPSpeakers Bureau: AstraZeneca, Novartis, Reliant Pharmaceuticals, Inc.Michael H. Davidson, M.D., FACCResearch Support: Abbott Laboratories, AstraZeneca Pharmaceuticals, Daiichi-Sankyo, Inc., Merck/Schering-Plough,Pfizer Laboratories, Reliant Pharmaceuticals, Inc., Roche Pharmaceuticals, sanofi-aventis, Takeda Pharmaceuticals;Speakers Bureau: Abbott Laboratories, AstraZeneca Pharmaceuticals, Daiichi-Sankyo Inc., Merck & Co. Inc., MerckSchering Plough, Pfizer Inc., Reliant Pharmaceuticals, Inc., Takeda Pharmaceuticals; Consultant: Abbott Laboratories,AstraZeneca Pharmaceuticals, Daiichi-Sankyo Inc., Merck & Co. Inc., Merck Schering-Plough, Pfizer Inc., ReliantPharmaceuticals, Inc., Takeda Pharmaceuticals.M. Sue Kirkman, M.D.Disclosed no conflict of interest.Dante S. LaRocca, Ph.D.Disclosed no conflict of interest.Stanley Schwartz, M.D., FACPSpeakers Bureau: Amylin, Eli Lilly, Merck & Co. Inc., Novo Nordisk, Inc., Pfizer Inc., sanofi-aventis, TakedaPharmaceuticals; Advisory Panel: Eli Lilly, Takeda Pharmaceuticals.Russell D. White, M.D.Speaker Bureau: <strong>American</strong> Academy of Family Physicians, The National Procedures <strong>Association</strong>, University of Missouri-Kansas City; Consultant: Novo Nordisk, Inc.4<strong>American</strong> <strong>Diabetes</strong> <strong>Association</strong> <strong>Self</strong>-<strong>Assessment</strong> <strong>Program</strong>


General Instructions for Completion of ADA-SAP Module 1 of ADA-SAP consists of the following components: Multiple Choice Questions (MCQs) <strong>Assessment</strong> ComponentThis consists of 75 clinical problem-solving MCQs. Your task is to select the one lettered option amongfour or five offered that BEST answers each MCQ. AppendixThis is a comprehensive summary of the recommendations of the ADA for diagnosis and managementof type 2 diabetes. It is based on peer-reviewed publications and is designed to assist you to completethe <strong>Assessment</strong> and Educational Components of ADA-SAP . Answer SheetAn answer sheet is provided for you to record your answers as you work through the MCQs component ofthe program. Refer to this answer sheet to assess your performance while reading the Critiques component. Educational (Learning) Critiques ComponentIN ORDER TO MAXIMIZE YOUR LEARNING EXPERIENCE IT IS IMPORTANT THAT YOU DONOT LOOK AT THE EDUCATIONAL CRITIQUES COMPONENT UNTIL AFTER YOU HAVECOMPLETED MARKING YOUR ANSWERS FOR THE MCQs ON THE ANSWER SHEET. Thecritiques component contains detailed explanations for the correct and incorrect answers for the MCQs basedon the most current peer-reviewed published information. Once you have read the Educational Critiques, thebibliographic references should be utilized as follow-up study for those MCQs which you answered incorrectly.The critiques are the teaching and learning component of ADA-SAP . They are to be used in combinationwith the MCQ assessment component to provide you with a positive, active learning experience.DO NOT CHANGE THE ANSWERS MARKED ON YOUR ANSWER SHEET WHILE READINGTHE CRITIQUES. THE INTENT OF ADA-SAP IS LEARNING NEW KNOWLEDGE ANDREINFORCING PREVIOUSLY LEARNED KNOWLEDGE. <strong>Program</strong> FeedbackAfter you have completed the MCQ and Critique components , you may email any comments about the contentor format of this <strong>Self</strong>-<strong>Assessment</strong> <strong>Program</strong> to info@proevalinc.com.<strong>American</strong> <strong>Diabetes</strong> <strong>Association</strong> <strong>Self</strong>-<strong>Assessment</strong> <strong>Program</strong>5


<strong>American</strong> <strong>Diabetes</strong> <strong>Association</strong><strong>Self</strong>-<strong>Assessment</strong> <strong>Program</strong>Multiple Choice Questions (MCQs)6<strong>American</strong> <strong>Diabetes</strong> <strong>Association</strong> <strong>Self</strong>-<strong>Assessment</strong> <strong>Program</strong>MCQs


Multiple Choice Questions (MCQs)Items 1–75DirectionsItems 1–75 consist of a question or an incomplete statement followed by three, four or five lettered options.Your task is to select the one BEST lettered option that answers each item. After you have selected the onelettered option that BEST answers each item, completely blacken the corresponding lettered circle for thatitem on Section A of the answer sheet. Please use only a #2 soft lead pencil.Complex Case Highlight (Items 1–10)A 35-year-old African-<strong>American</strong> man presents to you for his annual “physical exam.” He has no significantcomplaints but is concerned because his older sister was just diagnosed with type 2 diabetes. His father alsohad type 2 diabetes and died at age 54 of a myocardial infarction. He has two sons age 10 and 12, both 120%of ideal weight for their height. He is 6’0” and weighs 210 lbs (BMI 27 kg/m 2 ); blood pressure is 138/88 mm Hg.Current lab results for this patient are as follows:Total CholesterolTriglyceridesHDL-CLDL-CGlucose210 mg/dL150 mg/dL40 mg/dL140 mg/dL96 mg/dL Items 1–6For each numbered laboratory result (1–6), (if confirmed by repeat testing), select the one lettered diagnosis for thispatient’s problem (A, B, C, D) MOST likely associated with it. Each lettered diagnosis may be selected once, morethan once, or not at all.(A) Impaired fasting glucose (IFG).(B) Impaired glucose tolerance (IGT).(C) <strong>Diabetes</strong>.(D) None of the above.1. Fasting plasma glucose 120 mg/dL.2. 2-hour plasma glucose 190 mg/dL.3. Fasting plasma glucose 130 mg/dL.MCQs<strong>American</strong> <strong>Diabetes</strong> <strong>Association</strong> <strong>Self</strong>-<strong>Assessment</strong> <strong>Program</strong>7


4. 2-hour plasma glucose 210 mg/dL after a dose of 75 g of anhydrous glucose dissolved in water.5. Non-fasting glucose (he ate breakfast 4 hours before) 160 mg/dL.6. The same patient but now with a history of polyuria and a non-fasting glucose of 210 mg/dL.7. Which one of the following statements regarding this patient is CORRECT?(A) According to ADA guidelines, since his fasting glucose is 100 mg/dL, testing should be repeatedat 3-year intervals.(B) According to the NCEP ATP III guidelines, he is at his LDL goal for his risk category.(C) According to the JNC7, he does not require drug therapy for hypertension.(D) According to the ADA guidelines, an oral glucose tolerance test (OGTT) is recommended to potentiallydiagnose diabetes in patients with a strong family history.(E) According to ADA guidelines, his two sons should have their fasting plasma glucose measured andrepeated every 2 years.Upon the recommendation of your patient, his sister comes to see you for treatment of her diabetes. His sister is39 years old and was diagnosed with “diabetes” at a health fair in which she had a non-fasting glucose of 180 mg/dLapproximately 6 weeks ago. She has 5 children ages 10–20 years old. At birth, all her children weighed at least9 lbs. She has a history of increased blood pressure for the past 5 years and has tried to watch her salt intake andlose weight but takes no medication. For the past 6 weeks, she has restricted her caloric intake and started walkingaround the block at least 5 times per week. She believes she has lost a “few pounds” but complains of feeling“sluggish” and she urinates more than usual. Her labs are as follows:Total CholesterolTriglyceridesHDL-CLDL-CFasting Glucose180 mg/dL200 mg/dL38 mg/dL102 mg/dL170 mg/dLA1C 7.2%Creatinine1.1 mg/dLBUN 20ALT 48 (normal 40)AST 50 (normal 45)Height 5’8”WeightBlood Pressure170 lbs138/88 mm Hg8<strong>American</strong> <strong>Diabetes</strong> <strong>Association</strong> <strong>Self</strong>-<strong>Assessment</strong> <strong>Program</strong>MCQs


8. According to the ADA guidelines, which one of the following statements regarding the patient’s sisteris CORRECT?(A) Before initiating drug therapy, the patient should undergo an exercise stress test to rule out significantcoronary artery disease and, if normal, an aerobic exercise program should be prescribed for at least60 minutes/day 5 times per week.(B) Start a low-carbohydrate diet (restricting total carbohydrates to 130 g/day) to induce short-term weightloss and thereby improve glycemic control.(C) Start metformin 1000 mg/day in conjunction with a weight reduction meal plan and an exercise program.(D) Start therapy with a sulfonylurea because she has symptomatic hyperglycemia and this medication willresult in more rapid reduction in her elevated glucose level.(E) Start with a glitazone because she has a very low HDL (50 mg/dL) for a woman.9. In addition to treatment for hyperglycemia, which one of the following statements is CORRECT regardingthe management of her other cardiovascular risk factors according to AHA recommendations?(A) Since she has a systolic blood pressure of 130–139 mm Hg and a diastolic blood pressure of80–89 mm Hg and has failed lifestyle and behavior therapy, she should be treated with pharmacologicalagents that block the rennin-angiotensin system.(B) Since she is less than 40 years of age, statin therapy is not recommended.(C) Since she does not have overt CVD, her LDL goal is 130 mg/dL rather than 100 mg/dL.(D) For a women with diabetes, her triglyceride goal is 100 mg/dL and her HDL goal is 40 mg/dL.(E)A fibrate is recommended as the drug of first choice to treat her dyslipidemia because she has elevatedtriglycerides.10. For a patient with newly-diagnosed type 2 diabetes, which one of the following ancillary tests is recommended?(A) Urinary evaluation for microalbuminuria.(B) A dilated and comprehensive eye examination by an ophthalmologist or optometrist.(C) Screening for distal symmetric polyneuropathy.(D) Comprehensive foot exam.(E) All of the above. Items 11–12A 55-year-old African-<strong>American</strong> female school teacher whose mother and father both developed type 2 diabetesin their 50’s presents to you for an evaluation to determine her risk for developing diabetes mellitus. She has4 children and during the pregnancy of her fourth child, she developed gestational diabetes. She is a non-smokerand sedentary. She is 5’5”, weighs 200 lbs, and blood pressure is 128/78 mm Hg. Fasting labs are as follows:Total CholesterolTriglyceridesHDL-CLDL-CGlucose200 mg/dL100 mg/dL50 mg/dL130 mg/dL110 mg/dLMCQs<strong>American</strong> <strong>Diabetes</strong> <strong>Association</strong> <strong>Self</strong>-<strong>Assessment</strong> <strong>Program</strong>9


She takes no medications including aspirin. She brings with her a Personal Health Decisions report fromwww.diabetes.org/diabetesphd, which is an appraisal on the ADA website for assessing the risk of developingdiabetes, heart disease, or a stroke over the next 30 years.11. Based on the Personal Health Decisions (<strong>Diabetes</strong> PHD) appraisal, she has a 38% chance of developing type2 diabetes. If she loses 10% of her body weight (approximately 20 lbs), her risk of developing type 2 diabeteswould be which one of the following?(A) Modified to 10%.(B) Modified to 11–20%.(C) Modified to 21–30%.(D) Her risk remains 35% because she is still significantly overweight.(E) Due to her history of gestational diabetes, weight loss would be ineffective in modifying her riskof developing type 2 diabetes.12. According to the ADA Guidelines for the prevention or delay of type 2 diabetes, which one of the followingnext steps would be MOST appropriate?(A) Initiate therapy with metformin simultaneously with a weight reduction meal plan or exercise program.(B) Perform a 2-hour oral glucose tolerance test and only initiate metformin if her 2-hour post-prandialglucose is greater than 180 mg/dL.(C) Instruct her on a weight reduction meal plan with exercise. If she fails to lose weight and her fastingglucose remains below 126 mg/dL, medication remains inappropriate.(D) Instruct her on a weight reduction meal plan program with exercise and consider initiation of metforminin 6–12 months if she has not lost significant amounts of weight and her fasting glucose remains greaterthan 100 mg/dL.(E) Instruct the patient on a weight reduction meal plan with exercise with the addition of a weight lossmedication, such as orlistat. Item 13A 40-year-old Hispanic woman with newly-diagnosed type 2 diabetes mellitus has been on a weight reductionmeal plan for the past 3 months and has lost 5 lbs. She is on simvastatin 20 mg, enalapril 10 mg, and aspirin 81 mg.Her labs are as follows:Total CholesterolTriglyceridesHDL-CLDL-CGlucose160 mg/dL200 mg/dL40 mg/dL70 mg/dL180 mg/dLA1C 9.0%Height 5’1”Weight150 lbsWaist Circumference 37”Blood Pressure128/80 mm Hg10<strong>American</strong> <strong>Diabetes</strong> <strong>Association</strong> <strong>Self</strong>-<strong>Assessment</strong> <strong>Program</strong>MCQs


13. According to the <strong>American</strong> <strong>Diabetes</strong> <strong>Association</strong> (ADA) treatment algorithm for type 2 diabetes,the MOST appropriate treatment option for this patient is which one of the following?(A) Since she has lost 5 lbs, oral hypoglycemic agents should be withheld until weight loss treatmenthas been maximized.(B) Initiate metformin therapy as the first-line treatment for type 2 diabetes.(C) Since she has significant glucose elevation, a sulfonylurea should be initiated because these oralhypoglycemic agents are the most potent in treating hyperglycemia.(D) Because she has elevated triglyceride and low HDL levels, a thiazolidinedione (TZD) should be initiated.(E) She should receive treatment with sitagliptin because it has a low side-effect profile and is a category Bdrug for women of child-bearing potential. Items 14–18For each numbered clinical scenario or pharmacologic effect (14–18), select the one MOST appropriate letteredPPAR agonist treatment option associated (A, B, C, D, E). Each lettered PPAR agonist treatment option may beselected once, more than once, or not at all.(A) Fenofibrate.(B) Gemfibrozil.(C) Thiazolidinedione.(D) None of the above.(E) All of the above.14. Contraindicated in a man with cardiovascular disease and NYHA Class I heart failure.15. Inhibits statin glucuronidation leading to elevated statin levels.16. Dosage adjustment is necessary in patients with moderate to severe renal impairment.17. Not contraindicated in a woman with diabetes and fatty liver disease in whom the ALT is 75 U/L(normal 40) and the AST 46 U/L (normal 36).18. Causes a fall in the hematocrit. Items 19–23For each numbered physiological or pathophysiological effect (19–23), select the one MOST accurate letteredadipocytokine (A, B, C, D, E) associated with that effect. Each lettered adipocytokine may be selected once,more than once, or not at all.(A) Adiponectin.(B) TNF-α (tumor necrosis factor-alpha).(C) Free fatty acids.(D) Both TNF-α and free fatty acids.(E) PAI-1 (plasminogen activator inhibitor-1).MCQs<strong>American</strong> <strong>Diabetes</strong> <strong>Association</strong> <strong>Self</strong>-<strong>Assessment</strong> <strong>Program</strong>11


19. Increases insulin sensitivity.20. Inhibits monocyte adhesion to vascular wall endothelium.21. Causes hyperinsulinemia by interfering with hepatic insulin degradation.22. Contributes to insulin resistance.23. Localizes in the plaque where it is thought to promote coagulation.24. Which one of the following statements is CORRECT regarding the therapy of patients with type 2 diabetesmellitus with biguanides?(A) They will be the drug of choice in patients with compromised left ventricular function due to their lowrisk of hypoglycemia.(B) They should not be used in patients with significant renal impairment.(C) They should not be used in combination with insulin due to the increased risk for side-effects.(D) They lose efficacy when combined with a thiazolidinedione.(E) They are best tolerated and most effective when taken on an empty stomach.25. Which one of the following mechanisms MOST likely contributes to accelerated vascular disease associatedwith insulin resistance?(A) The direct effect of elevated insulin levels on the vessel wall.(B) A pro-inflammatory state.(C) Accelerated fatty acid oxidation in vascular smooth muscle cells.(D) Reduced plasminogen activator inhibitor-1 (PAI-1) levels.(E) Reduced homocysteine levels.26. Which one of the following statements is CORRECT regarding home monitoring of blood glucose?(A) It is only useful for patients on insulin.(B) In the most aggressively managed patients it should be done every 6 hours.(C) It is generally not necessary in patients with type 2 diabetes mellitus.(D) Measuring a fasting blood glucose is most helpful for adjusting a basal insulin dose.(E) None of the above.12<strong>American</strong> <strong>Diabetes</strong> <strong>Association</strong> <strong>Self</strong>-<strong>Assessment</strong> <strong>Program</strong>MCQs


Items 27–31For each numbered oral hypoglycemic agent (27–31), select the one lettered mechanism of action (A, B, C, D, E)which is MOST important for its efficacy in lowering blood glucose. Each lettered mechanism of action may beselected only once.(A) Increases insulin secretion.(B) Delays glucose absorption in the intestine.(C) Suppresses glucose production in the liver.(D) Increases glucose uptake into muscle.(E) Inhibits the enzyme that degrades GLP-1 (glucagon-like protein-1).27. Sulfonylureas.28. Thiazolidinediones.29. Biguanides.30. Alpha-glucosidase inhibitors.31. Sitagliptin. Items 32–35For each numbered undesirable side-effect (32–35), select the one lettered class of oral hypoglycemic agent(A, B, C, D, E) MOST closely associated with it. Each lettered class of oral hypoglycemic agent may beselected once, more than once, or not at all.(A) Sulfonylureas.(B) Thiazolidinediones.(C) Biguanides.(D) Alpha-glucosidase inhibitors.(E) None of the above.32. Increased fluid retention and edema.33. Nausea; lactic acidosis.34. Hypoglycemia.35. Gas, bloating.MCQs<strong>American</strong> <strong>Diabetes</strong> <strong>Association</strong> <strong>Self</strong>-<strong>Assessment</strong> <strong>Program</strong>13


36. A female patient with type 2 diabetes mellitus, weighing 300 lbs, asks you about treatment optionsbut is afraid of using medication for the treatment of diabetes. Which one of the following is the MOSTeffective method of achieving significant weight loss in patients with diabetes mellitus without usingantihyperglycemia medications?(A) Diets emphasizing fewer calories.(B) Increased physical activity.(C) Diets emphasizing a reduction in saturated fats.(D) Bariatric surgery.(E) Medications for weight loss.37. An overweight man with type 2 diabetes mellitus and peripheral neuropathy asks you about starting anexercise program to lose weight. Which one of the following is NOT a beneficial effect of incorporatingincreased physical activity and weight loss into the regimen of this patient?(A) Reduction in blood glucose.(B) Reduction in blood pressure.(C) Improvement in lipid profile.(D) Improved sense of control over the illness.(E) Will achieve less injuries related to neuropathy.38. Metformin is often selected when starting a patient with newly-diagnosed diabetes on medication for the firsttime. Which one of the following is NOT an advantage with metformin over other medications?(A) It rarely causes hypoglycemia.(B) It rarely induces weight gain.(C) It is associated with no serious side-effects.(D) It is generally inexpensive.(E) When dosed effectively it will usually lower A1C more than 1%.39. Sulfonylureas are frequently prescribed as part of the regimen for patients with type 2 diabetes.Which one of the following concerns about sulfonylureas is INCORRECT?(A) Have a tendency to cause weight gain.(B) May cause hypoglycemia, especially the longer-acting agents.(C) Have been consistently shown to increase mortality from cardiovascular disease.(D) May interact with other sulfa drugs or glinides to increase risk of hypoglycemia.(E) May cause a rash.14<strong>American</strong> <strong>Diabetes</strong> <strong>Association</strong> <strong>Self</strong>-<strong>Assessment</strong> <strong>Program</strong>MCQs


40. The glinides (e.g., repaglinide and nateglinide) are a recommended substitute for sulfonylureas for useas monotherapy in patients with type 2 diabetes mellitus with which one of the following?(A) Congestive heart failure.(B) Liver disease.(C) Renal insufficiency.(D) Associated hypertriglyceridemia. Items 41–45For each numbered anti-hyperglycemic agent (41–45), select one lettered primary mechanism of action(A, B, C, D, E) associated with it. Each lettered primary mechanism of action may be selected only once.(A) Reduces gluconeogenesis in the liver.(B) Stimulates post-prandial insulin secretion.(C) Delays the absorption of carbohydrates in the gut.(D) Enhances insulin sensitivity in the peripheral tissues.(E) Inhibits the breakdown of incretins.41. Metformin.42. Glitazones.43. Alpha-glucosidase inhibitors.44. Glinides.45. Sitagliptin. Items 46–48A 50-year-old woman with type 2 diabetes mellitus is seen in the office with a blood pressure of 150/90 mm Hgand the lipid profile shown below. She has a normal amount of albumin in her urine and normal renal function.Total CholesterolTriglyceridesHDL-CLDL-C215 mg/dL120 mg/dL51 mg/dL140 mg/dLMCQs<strong>American</strong> <strong>Diabetes</strong> <strong>Association</strong> <strong>Self</strong>-<strong>Assessment</strong> <strong>Program</strong>15


46. If her doctor elects to start her on medication for her blood pressure, which one of the following classesof antihypertensive agents would be BEST to reduce her chance of progression to microalbuminuria?(A) Alpha blockers.(B) Diuretics.(C) ACE inhibitors.(D) Beta blockers.(E) Calcium channel blockers.47. She is reluctant to take a statin. Which one of the following statements is INCORRECT regarding statinsand the prevention of CVD?(A) Statins have been shown to reduce the chances of a CV event in individuals that have no knownCVD (primary prevention).(B) All the large published studies which evaluated statins’ ability to prevent all cause mortality have showna benefit.(C) Statins have been shown to reduce the chances of a second CV event in individuals that have knownCVD (secondary prevention).(D) Statins have been shown to reduce the chances of having a CV event whether the LDL level is116 mg/dL or 116 mg/dL(E) The <strong>American</strong> <strong>Diabetes</strong> <strong>Association</strong> recommends all patients with type 2 diabetes mellitus reduceLDL levels to 100 mg/dL or lower.48. An alpha-glucosidase inhibitor, such as acarbose would be LEAST appropriate for the treatment of a patientwith type 2 diabetes mellitus and which one of the following clinical characteristics?(A) A1C level of 7.5 mg/dL with kidney disease.(B) A1C level of 7.5 mg/dL with liver disease.(C) A1C level of 8.0 mg/dL with a tendency toward hypoglycemia.(D) A1C level of 12.0 mg/dL with low tolerance for side-effects.(E) A1C level of 7.2 mg/dL with concerns about systemic side-effects.49. Insulin should be considered, along with lifestyle modifications, as the initial therapy in a newly-diagnosedpatient with type 2 diabetes mellitus in all of the following situations EXCEPT:(A) Fasting blood glucose level 250 mg/dL.(B) A1C 10%.(C) BMI 27 kg/m 2 .(D) Presence of ketonuria.(E) Symptomatic hyperglycemia (polyuria, polydypsia).16<strong>American</strong> <strong>Diabetes</strong> <strong>Association</strong> <strong>Self</strong>-<strong>Assessment</strong> <strong>Program</strong>MCQs


50. A 43-year-old overweight hypertensive woman is newly-diagnosed with type 2 diabetes mellitus, with a A1Cof 8.5%. Which one of the following is the BEST choice for initial therapy in addition to weight reduction?(A) Metformin.(B) A sulfonylurea.(C) A glitazone.(D) Inhaled insulin.(E) Sitagliptin.51. When initiating insulin therapy in a patient with type 2 diabetes, which one of the following approachesis recommended?(A) Start with a bedtime dose of an intermediate-acting form or a once daily, long-acting form.(B) Start with a 2-shot regimen of a long-acting form.(C) Start with a 3-shot regimen of a short-acting form.(D) Start with a higher dose and titrate down.(E) Make dosing changes infrequently to give time for the patient to adjust to the dose.52. A 48-year-old man with type 2 diabetes, treated with a basal dose of long-acting insulin and oral medications,is struggling to control his post-prandial blood glucose. When adding rapid-acting insulin with meals to abasal insulin dosage, it is important to remember which one of the following additional adjustments?(A) Reduce the basal insulin dose to avoid hypoglycemia.(B) Discontinue any insulin secretogogue, like a sulfonylurea or glinide.(C) Increase the dose of the insulin secretogogues to enhance the effect of the injected insulin.(D) Be sure to add an ACE inhibitor to improve renal function.(E) Reduce the total carbohydrate intake by 30% to avoid hyperglycemia. Items 53–54A 51-year-old woman with type 2 diabetes has been treated for 3 months with lifestyle modificationsand metformin at 1000 mg b.i.d., but her A1C level is still 7%.53. Which one of the following steps would be LEAST appropriate for her at this time?(A) Add a glitazone (rosiglitazone or pioglitazone).(B) Add glipizide.(C) Add a single dose of basal insulin.(D) Increase the metformin to 1000 mg t.i.d.(E) Add glyburide.MCQs<strong>American</strong> <strong>Diabetes</strong> <strong>Association</strong> <strong>Self</strong>-<strong>Assessment</strong> <strong>Program</strong>17


54. The patient is reluctant to intensify her therapy because she feels fine. In discussing the benefits of reducingher A1C level, you consider how much benefit she can derive from additional reduction. According tothe United Kingdom Prospective <strong>Diabetes</strong> Study (UKPDS), for every 1% reduction in A1C level in theaverage patient with type 2 diabetes, which one of the following results would NOT be expected?(A) A 21% decrease in any diabetes-related endpoint.(B) A 14% decrease in the risk of myocardial infarction.(C) A 12% decrease in the risk of stroke.(D) A 37 % decrease in the risk of microvascular disease.(E) A 42% decrease in the risk of heart failure.55. Which one of the following vaccines is recommended for all patients with diabetes between the agesof 18–64 years?(A) Meningococcal.(B) Pneumococcal once and yearly influenza.(C) Hepatitis A.(D) Hepatitis B.(E) Pneumococcal every 5 years and influenza yearly.56. On average, patients with type 2 diabetes mellitus have lost approximately what percentage of their pancreaticbeta-cell (β-cell) function by the time their diabetes is diagnosed?(A) 20%.(B) 35%.(C) 50%.(D) 70%.(E) 90%.57. Which one of the following is NOT commonly associated with hypoglycemia?(A) Anxiety.(B) Tachycardia.(C) Shakiness.(D) Death.(E) Diaphoresis.18<strong>American</strong> <strong>Diabetes</strong> <strong>Association</strong> <strong>Self</strong>-<strong>Assessment</strong> <strong>Program</strong>MCQs


Items 58–59The pathophysiology of type 2 diabetes mellitus requires both insulin resistance and pancreatic β-cell dysfunction.The major targets for insulin are the liver, skeletal muscle, and adipose tissue.58. Which one of the following statements is INCORRECT regarding insulin resistance in patients withtype 2 diabetes mellitus?(A) The normal suppression of gluconeogenesis by insulin in the liver is impaired.(B) The normal uptake of glucose by skeletal muscle is impaired.(C) The normal glucose disposal, which includes glucose oxidation and glycogen synthesis, by skeletalmuscle, is impaired.(D) Excess free fatty acids (FFAs) impair β-cell function.(E) The normal degradation of triglycerides into FFAs by adipose tissue is impaired.59. Which one of the following is NOT a major factor in pancreatic β-cell dysfunction in patients withtype 2 diabetes mellitus?(A) Environmental factors.(B) Genetic factors.(C) Lipotoxicity.(D) Glucotoxicity.(E) Insulin toxicity.60. Which one of the following conditions depicted pictorially is NOT associated with type 2 diabetes mellitus?(A)(B)(C)(D)MCQs<strong>American</strong> <strong>Diabetes</strong> <strong>Association</strong> <strong>Self</strong>-<strong>Assessment</strong> <strong>Program</strong>19


Management of a patient with type 2 diabetes (Items 61–71)A 28-year-old African-<strong>American</strong> man presented to his primary care physician’s office with complaints of knee pain.In college, he played guard on the football team. He had anterior cruciate ligament surgery after college, stoppedregular exercise, and gradually increased weight.Past History: no other surgery.Family History: positive for type 2 diabetes mellitus, hypertension.Social History: negative for smoking, no drug abuse, occasional alcohol intake.Review of symptoms: non-contributory.TriglyceridesHDL-CLDL-CPost-prandial Glucoses (PPG)152 mg/dL38 mg/dL110 mg/dL144 mg/dL & 154 mg/dLA1C 5.6%Blood PressureHeart Rate130/82 mm Hg76 BPMRespirations 14BMI 34 kg/m 2FundiPulmonary, cardiac,abdominal & neurological examsnormalnormal61. The patient asks how to prevent overt diabetes. Which one of the following therapies has NOT been provento prevent diabetes in patients with impaired glucose tolerance?(A) Lifestyle changes.(B) Metformin.(C) Acarbose.(D) Rosiglitazone.(E) Ramipril.62. Diet should never be forgotten as part of the care of patients with diabetes. The optimal mix of macronutrientsis not clear; in fact, a diet consisting of foods which the patient usually eats, modified to fit basic principles,will ensure compliance. In a patient, such as this 28-year-old African-<strong>American</strong> man with impaired glucosetolerance or type 2 diabetes, which one of the following diets is NOT recommended?(A) A diet that includes sugar alcohols and non-nutritive sweeteners in FDA-approved quantities.(B) A diet that includes fruits, vegetables, fiber, whole grain, legumes and low fat meals.(C) A diet that includes fiber.(D) A diet that includes 20 g of carbohydrate foods daily.(E) A diet that includes controlled quantities of sucrose-containing foods.20<strong>American</strong> <strong>Diabetes</strong> <strong>Association</strong> <strong>Self</strong>-<strong>Assessment</strong> <strong>Program</strong>MCQs


63. Which one of the following is a characteristic seen in some African-<strong>American</strong>s with new-onset type 2 diabetes?(A) Ability to stop insulin therapy soon after presentation with diabetic ketoacidosis (DKA).(B) A particular responsiveness to a thiazolidinedione (TZD).(C) Commonly have anti-islet cell antibodies.(D) Low incidence of family history of diabetes.(E) Associated with sickle cell trait.64. A generally acceptable ideal goal of treatment for this patient might include each of the following EXCEPT:(A) A1C 7%.(B) A1C 6% without undue hypoglycemia.(C) Fasting blood glucose 140 mg/dL.(D) Peak post-prandial blood glucose 180 mg/dL.65. This patient’s A1C level on lifestyle therapy alone was maintained at ~5.8% for about a year, but graduallyrose to 6.5%, and on return to the practice was 7.2%. It was elected to prescribe metformin. According toADA guidelines, metformin should be started at the time of diagnosis because of which one of the following?(A) Effectiveness in decreasing A1C.(B) Low cost.(C) Acceptable low risk of side-effects.(D) Proven outcome benefits.(E) All of the above.66. Which one of the following is NOT an effect of therapy with metformin?(A) It reduces hepatic glucose output.(B) It reduces hepatic gluconeogenesis.(C) It reduces insulin levels.(D) It reduces fasting blood glucose levels.(E) It increases C-peptide levels.67. The patient did well for about 2 years on metformin. However the AIC again increased to 7.2% andpioglitazone was added to metformin. His A1C decreased to 6.4%. However, his A1C increased againand it was elected to add a third agent when it reached 7.1%. Which one of the following is NOT a logicalchoice for a third drug added to this patient’s treatment?(A) Glipizide.(B) Rosiglitazone.(C) Glargine.(D) Exenatide.(E) Sitagliptin.MCQs<strong>American</strong> <strong>Diabetes</strong> <strong>Association</strong> <strong>Self</strong>-<strong>Assessment</strong> <strong>Program</strong>21


68. Which one of the following is NOT an abnormality resulting from increased visceral fat in patientswith the metabolic syndrome?(A) Increased myocyte insulin resistance.(B) Increased liver insulin resistance.(C) Increased adiponectin level.(D) Increased likelihood for the onset of type 2 diabetes.(E) Decreased β-cell function.69. Which one of the following statements is INCORRECT regarding post-prandial glycemia?(A) It is the earliest detectable glycemic abnormality in the majority of patients.(B) Convincing data exists that treating it will reduce cardiovascular outcomes or other complicationsof diabetes.(C) It contributes to the A1C level of 6.5% to a greater extent than does fasting blood glucose.(D) It is associated with an increased risk of microvascular complications.(E) It is associated with an increased risk of macrovascular complications.70. Primary data supporting the use of analog insulin does NOT include which one of the following?(A) It mimics normal physiology.(B) It decreases overnight hypoglycemia.(C) It reduces A1C level more than does human insulin.(D) Patient lifestyle will control the disease therapy rather than the disease therapy controlling the lifestyle.(E) It decreases late post-prandial hypoglycemia.71. Which one of the following is NOT a likely mechanism for complications in diabetes mellitus?(A) Increased pentose-phosphate shunt flux.(B) Increased flux in aldose reductase pathway.(C) Increased advanced glycosylation end product production.(D) Increased hexosamine pathway flux.(E) Increased oxidative stress in mitochondria.72. Which one of the following statements about treatments of hypertension in patients with type 2 diabetesis INCORRECT?(A) Aim for blood pressure under 130/80 mm Hg.(B) Include an ACE inhibitor or an ARB if possible.(C) Often requires combination therapy.(D) Must absolutely contraindicate hydrochlorothiazide (HCTZ) because it may increase blood glucose.(E) Beta blockade may increase the risk of hypoglycemic unawareness.22<strong>American</strong> <strong>Diabetes</strong> <strong>Association</strong> <strong>Self</strong>-<strong>Assessment</strong> <strong>Program</strong>MCQs


Items 73–75A logical choice of combination therapy with metformin should depend on individual characteristics of patientswith type 2 diabetes. For each numbered patient being treated with metformin (73–75), select the one letteredadditional agent in combination therapy (A, B, C) most appropriate for that patient. Each lettered additional agentin combination therapy may be selected only once.(A)(B)(C)Insulin.Exenatide.Sitagliptin.73. A patient with a BMI 35 kg/m 2 .74. A patient with A1C 9.6%.75. A patient who is a long-distance truck driver and fears injections.MCQs<strong>American</strong> <strong>Diabetes</strong> <strong>Association</strong> <strong>Self</strong>-<strong>Assessment</strong> <strong>Program</strong>23


Notes24<strong>American</strong> <strong>Diabetes</strong> <strong>Association</strong> <strong>Self</strong>-<strong>Assessment</strong> <strong>Program</strong>


AppendixI. <strong>American</strong> <strong>Diabetes</strong> <strong>Association</strong> Guidelinesand Treatment Algorithms At-a-GlanceAppendix<strong>American</strong> <strong>Diabetes</strong> <strong>Association</strong> Guidelines and Treatment Algorithms At-A-Glance25


Definition and Description of <strong>Diabetes</strong> Mellitus<strong>Diabetes</strong> mellitus is a group of metabolic diseases characterized by hyperglycemia resulting from defects in insulinsecretion, insulin action, or both. The chronic hyperglycemia of diabetes is associated with long-term damage,dysfunction, and failure of various organs, especially the eyes, kidneys, nerves, heart, and blood vessels.I. Diagnosis of impaired glucose tolerance (IGT) and impaired fasting glucose (IFG)This group is defined as having fasting plasma glucose (FPG) levels 100 mg/dL (5.6 mmol/L) but126 mg/dL (7.0 mmol/L) or 2-h values in the oral glucose tolerance test (OGTT) of 140 mg/dL (7.8 mmol/L)but 200 mg/dL (11.1 mmol/L). Thus, the categories of FPG values are as follows: FPG 100 mg/dL (5.6 mmol/L) normal fasting glucose; FPG 100–125 mg/dL (5.6–6.9 mmol/L) = IFG; FPG 126 mg/dL (7.0 mmol/L) provisional diagnosis of diabetes (the diagnosis must be confirmed,as described below).The corresponding categories when the OGTT is used are the following: 2-h post load glucose 140 mg/dL (7.8 mmol/L) normal glucose tolerance; 2-h post load glucose 140–199 mg/dL (7.8–11.1 mmol/L) = IGT; 2-h post load glucose 200 mg/dL (11.1 mmol/L) provisional diagnosis of diabetes (the diagnosis mustbe confirmed, as described below).Patients with IFG and/or IGT are now referred to as having “pre-diabetes” indicating the relatively high riskfor development of diabetes in these patients.Table 1. Treatment recommendation for individuals with IFG, IGT, or both.PopulationIFG or IGT.Individuals with IFG and IGT and any of the following: 60 years of age. BMI 35 kg/m 2 . Family history of diabetes in first-degree relatives. Elevated triglycerides. Reduced HDL-C. Hypertension. A1C 6.0%.TreatmentLifestyle modification (i.e., 5–10% weight loss and moderateintensity physical activity 30 min/day).Lifestyle modification (as above) and/or metformin*.* Metformin 850 mg twice/day.26 <strong>American</strong> <strong>Diabetes</strong> <strong>Association</strong> Guidelines and Treatment Algorithms At-A-Glance Appendix


II. Diagnosis of type 2 diabetesTable 2. Criteria for testing for diabetes in asymptomatic adult individuals.1. Testing for diabetes should be considered in all individuals at age 45 years and above, particularly in those with aBMI 25 kg/m 2 *, and, if normal, should be repeated at 3-year intervals.2. Testing should be considered at a younger age or be carried out more frequently in individuals who are overweight(BMI 25 kg/m 2 *) and have additional risk factors: Are habitually physically inactive. Have a first-degree relative with diabetes. Are members of a high-risk ethnic population (e.g., African-<strong>American</strong>, Latino, Native <strong>American</strong>, Asian <strong>American</strong>,Pacific Islander). Have delivered a baby weighing 9 lbs or have been diagnosed with GDM. Are hypertensive (140/90 mm Hg). Have an HDL-C level 35 mg/dL (0.90 mmol/L) and/or a triglyceride level 250 mg/dL (2.82 mmol/L). Have PCOS (polycystic ovary syndrome). On previous testing, had IGT or IFG. Have other clinical conditions associated with insulin resistance (e.g., PCOS or acanthosis nigricans). Have a history of vascular disease.* May not be correct for all ethnic groups.Table 3. Testing for type 2 diabetes in children.Criteria Overweight (BMI 85th percentile for age and sex, weight for height 85th percentile, or weight 120%of ideal for height).Plus any two of the following risk factors: Family history of type 2 diabetes in first- or second- degree relative. Race/ethnicity (Native <strong>American</strong>, African-<strong>American</strong>, Latino, Asian <strong>American</strong>, Pacific Islander). Signs of insulin resistance or conditions associated with insulin resistance (acanthosis nigricans, hypertension,dyslipidemia, or PCOS). Maternal history of diabetes or GDM.Age of initiation: age 10 years or at onset of puberty, if puberty occurs at a younger age.Frequency: every 2 years.Test: FPG preferred.Clinical judgment should be used to test for diabetes in high-risk patients who do not meet these criteria.Table 4. Criteria for the diagnosis of diabetes mellitus.1. Symptoms of diabetes plus casual plasma glucose concentration 200 mg/dL (11.1 mmol/L).1). Casual is defined as any time of day without regard to time since last meal. The classic symptoms of diabetes includepolyuria, polydipsia, and unexplained weight loss.OR2. FPG 126 mg/dL (7.0 mmol/L). Fasting is defined as no caloric intake for at least 8 hours.OR3. 2-h post load glucose 200 mg/dL (11.1 mmol/L) during an OGTT. The test should be performed as described by theWorld Health Organization, using a glucose load containing the equivalent of 75 g anhydrous glucose dissolved in water.In the absence of unequivocal hyperglycemia, these criteria should be confirmed by repeat testing on a different day. The third measure (OGTT) is notrecommended for routine clinical use.Appendix<strong>American</strong> <strong>Diabetes</strong> <strong>Association</strong> Guidelines and Treatment Algorithms At-A-Glance27


III. Summary of recommendations for adults with diabetes.Glycemic ControlA1CPreprandial capillary plasma glucosePeak post-prandial capillary plasma glucose †Blood pressure7.0%*90–130 mg/dL (5.0–7.2 mmol/L)180 mg/dL (10.0 mmol/L)130/80 mm HgLipids ‡LDL-CTriglycerides* Referenced to a non-diabetic range of 4.0–6.0% using a DCCT-based assay.† Post-prandial glucose measurements should be made 1–2 hours after the beginning of the meal, generally peak levels in patients with diabetes.‡ Current NCEP/ATP III guidelines suggest that in patients with triglycerides 200 mg/dL, the “non-HDL-C” (total cholesterol minus HDL) be utilized.The goal is 130 mg/dL (121).§ For women, it has been suggested that the HDL-C goal be increased by 10 mg/dL.Table 5. Detailed diabetes history and evaluation.Past medical history should include: Prior glucose values including A1C, symptoms, treatments, and history of self-monitoring. History of complications: hypoglycemia and/or ketoacidosis – including frequency, severity, and causes. <strong>Self</strong>-management, self-monitoring education, attitudes, health beliefs. Mood disorder. Prior or current infections, particularly skin, foot, dental, and genitourinary. Current microvascular symptoms and complications (eye, kidney, nerves). Current macrovascular symptoms and complications (evaluation of cardiovascular risk factors and cardiovascular disease). Sexual dysfunction. Review of past/present lifestyle modification Eating patterns Smoking history Diet history Alcohol history Weight history Controlled substances Exercise history Other medications – evaluate for effect on blood glucose. Family history of diabetes and CVD.Referrals100 mg/dL (2.6 mmol/L)150 mg/dL (1.7 mmol/L)HDL-C 40 mg/dL (1.0 mmol/L) §Key concepts in setting glycemic goals: A1C is the primary target for glycemic control. Goals should be individualized. Certain populations (children, pregnant women, and elderly) require special considerations. More stringent glycemic goals (i.e., a normal A1C, 6%) may further reduce complications at the costof increased risk of hypoglycemia. Less intensive glycemic goals may be indicated in patients with severe or frequent hypoglycemia. Post-prandial glucose may be targeted if A1C goals are not met despite reaching preprandial glucose goals. Eye exam. Family planning for women of reproductive age. Dietitian for medical nutrition therapy (MNT). <strong>Diabetes</strong> educator. Foot specialist. Dentist.Adapted from <strong>American</strong> <strong>Diabetes</strong> <strong>Association</strong> Clinical Practice Recommendations, 2006. <strong>Diabetes</strong> Care 2006;29(suppl. 1):S9.28 <strong>American</strong> <strong>Diabetes</strong> <strong>Association</strong> Guidelines and Treatment Algorithms At-A-Glance Appendix


Table 6. Follow-up testing recommendations with diabetes.Potential Complications Test Frequency of EvaluationHyperglycemia A1C Measure A1C 2 times/year if at goal,quarterly if therapy changed, or not at goalFasting blood glucoseVaries depending on prescription regime<strong>Self</strong>-monitoring of blood glucoseFrequency/timing should be individualizedto facilitate reaching goalsHypertension Blood pressure Measure at every visitAutonomic Neuropathy Sitting and standing blood pressure Appropriate patients/each visit(Orthostatic Hypotension)Dyslipidemia Lipid panel Test annually – More often if notat goal or when changing therapyFoot Care Quantitative somato-sensory threshold Perform a visual inspection at eachtest using Semmes-Weinstein 5.07 (10 g) routine visit for people with neuropathy.monofilament, tuning fork, palpation, andvisual inspectionAnnual comprehensive foot examNephropathy Serum creatinine AnnuallyAlbumin/Creatinine ratio(Random spot urine)Type 2 diabetes mellitus – Annual screeningType 1 diabetes mellitus – Annual screeningwith duration of diabetes 5 yearsType 1 & 2 diabetes mellitus – 2 of 3specimens within a 3–6 month periodshould be abnormal before consideringa new diagnosisRetinopathy Comprehensive eye exam Type 2 diabetes mellitus – Annual dilated &comprehensive examType 1 diabetes mellitus – Initial dilated &comprehensive exam within 5 years of onset;annual exams thereafterPeriodontal disease (common Dental evaluation Annuallyin type 2 diabetes mellitus)Influenza and pneumonia Immunization Annual influenza vaccinationAdapted from <strong>American</strong> <strong>Diabetes</strong> <strong>Association</strong> Clinical Practice Recommendations, 2006. <strong>Diabetes</strong> Care 2006;29(suppl. 1):S4–42.Appendix<strong>American</strong> <strong>Diabetes</strong> <strong>Association</strong> Guidelines and Treatment Algorithms At-A-Glance29


Table 7. Drug choices based on drug & patient characteristicsMetforminThiazolidinedione SulfonylureaRepaglinide/Nateglinide *ExenatideSitagliptin(Dpp-4inhibtor)Acarbose/MiglitolDosingfrequency2–3x daily Piogli- Rosiglitazone:tazone:1x daily 2x dailyGlimepramide:1x dailyGlipizide/Glyburide:2x daily* 1 witheach mealInject beforebreakfastand dinner1x daily3x dailywith mealsSpeed of actionFBS/ppgWeeks/weeks Weeks Day Day Day Day MonthsEfficacyPost-prandialFBSHgA1Concern/side-effectsHypoglycemicrisk whenused aloneMonitor forxxxxxxxxGI upset, lacticacidosis: avoid inpatients with CHF,liver disease, acuteillness, otheracidosis, Cr >1.4,hospitalizedpatients and hold2 days afterIV contrast untilCreatinine stablexxxxxxxxEdemaAvoid with no saltdiet & avoid NSAIDChannel blocker,COX 2 inhibitorsCHF 1%xxxxxxxxxxxxxxxxxxxxxwt gain +++ wt gain ++/+ Nausea –Avoid by stopeating whenfeel fullNone None + + + + +/ + None None NoneCreatinine &creatinine clearanceEdema/Heart FailureHypoglycemiaxxxxHypoglycemia Hypoglcemiaif use withSU/insulinxxxxxNoneHypglocemiaif use withSU/insulinWeight Ø OR NC ≠ OR NC ≠ ≠ Decrease Neutral NCLipidsHDLLDLTGNCØ ØØ Ø≠≠Ø≠≠DecreasebothNCNCØ OR NC Decrease DecreaseInsulin levels Ø Ø ≠ (less with glim.) ≠ GlucosedependentriseElderlyPreg risk in pts.with PolycysticOvary SyndromeAVOID if >70 years(because Øcreatinineclearance) avoidCr decrease


Bibliography1. Byetta Prescribing Information. February 2007. www.byetta.com. Accessed March 2007.2. Fineman MS, et al: Effect on glycemic control of exenatide (synthetic exendin-4) additive to existing metformin and/or sulfonylureatreatment in patients with type 2 diabetes. <strong>Diabetes</strong> Care 2003;26(8):2370–7.3. FDA website. www.fda.gov/medwatch. Accessed June 2005.4. Buse JB, et al. Effects of exenatide (exendin-4) on glycemic control over 30 weeks in sulfonylurea-treated patients with type 2 diabetes.<strong>Diabetes</strong> Care 2004;27(11):2628–35.5. Triplitt CL, et al: <strong>Diabetes</strong> Mellitus. In Dipro J, et al(eds). Pharmacotherapy: Pathophysiologic approach. 6th ed.New York, NY: McGraw-Hill Companies. 2005;1333–13676. The <strong>American</strong> <strong>Association</strong> of Clinical Endocrinologists medical guidelines for the management of diabetes mellitus:The AACE system of intensive diabetes self-management 2002 update. Endocrine Practice 2002;8(Suppl. 1):40–65.7. <strong>American</strong> <strong>Diabetes</strong> <strong>Association</strong>. Diagnosis and classification of diabetes mellitus. <strong>Diabetes</strong> Care 2005;28(Suppl. 1):S37–S42.8. <strong>American</strong> <strong>Diabetes</strong> <strong>Association</strong>. Standards of medical care in diabetes. <strong>Diabetes</strong> Care 2005;28(Suppl. 1):S4–S36.9. Poulsen MK, et al: The combined effect of triple therapy with rosiglitazone, metformin, and insulin aspart in type 2 diabetic patients.<strong>Diabetes</strong> Care 2003;26(12):3273–3279.10. Dailey GE, et al: Glycemic control with glyburide/metformin tablets in combination with rosiglitazone in patients with type 2 diabetes:a randomized, double-blind trial. Am J Med 2004;116:223–229.11. DeFronzo RA, et al: Effects of exenatide (exendin-4) on glycemic control and weight over 30 weeks in metformin treated patientswith type 2 diabetes. <strong>Diabetes</strong> Care 2005;28(5):1092–1100.12. Kendall DM, et al: Effects of exenatide (exendin-4) on glycemic control over 30 weeks in patients with type 2 diabetes treated withmetformin and a sulfonylurea. <strong>Diabetes</strong> Care 2005;28(5):1083–1091.13. Klein S, et al: Weight management through lifestyle modification for the prevention and management of type 2 diabetes: rationaland strategies. <strong>Diabetes</strong> Care 2004;27(8):2067–2073.14. Chobanian AV, et al: Seventh Report of the Joint National Committee on prevention, detection, evaluation and treatment of highblood pressure. Hypertension 2003;42:1206–1252.15. Heine RJ, et al: Exenatide versus insulin glargine in patients with suboptimally controlled type 2 diabetes. Annals of Internal Medicine2005;143:559–569.16. Nathan DM, et al: Management of hyperglycemia in type 2 diabetes: A consensus algorithm for the initiation and adjustment oftherapy – A consensus statement from the <strong>American</strong> <strong>Diabetes</strong> <strong>Association</strong> and the European <strong>Association</strong> for the Study of <strong>Diabetes</strong>.<strong>Diabetes</strong> Care 2006;29(8)1963–1972.32 <strong>American</strong> <strong>Diabetes</strong> <strong>Association</strong> Guidelines and Treatment Algorithms At-A-Glance Appendix


<strong>American</strong> <strong>Diabetes</strong> <strong>Association</strong><strong>Self</strong>-<strong>Assessment</strong> <strong>Program</strong>Educational CritiquesIN ORDER TO MAXIMIZE YOUR LEARNING EXPERIENCE IT IS IMPORTANT THAT YOUDO NOT LOOK AT THE EDUCATIONAL CRITIQUES UNTIL AFTER YOU HAVE COMPLETEDMARKING YOUR ANSWERS FOR THE MCQs ON THE ANSWER SHEET.DO NOT CHANGE THE ANSWERS MARKED ON YOUR ANSWER SHEET WHILE READINGTHE CRITIQUES. THE INTENT OF ADA-SAP IS LEARNING NEW KNOWLEDGE ANDREINFORCING PREVIOUSLY LEARNED KNOWLEDGE. THERE IS NO PASS-FAIL SCORE.Educational (Learning) CritiquesNational Lipid <strong>Association</strong> <strong>Self</strong>-Study Module33


Educational (Learning) CritiquesItems 1–75Items 1–6Answers 1 (A); 2 (B); 3 (C); 4 (C); 5 (D); 6 (C)The ADA has established three criteria for the diagnosis of diabetes:1. Symptoms of diabetes and a casual plasma glucose 200 mg/dL (11.1 mmol/L). Casual is defined as any timewithout regard to time since last meal. The classic symptoms of diabetes include polyuria, polydipsia, andunexplained weight loss, OR2. Fasting plasma glucose (FPG) 126 mg/dL (7.0 mmol/L). Fasting is defined as no caloric intake for at least8 hours, OR3. 2-hour plasma glucose 200 mg/dL (11.1 mmol/L) during an oral glucose tolerance test (OGTT). Thetest should be performed as described by the World Health Organization, using a glucose load containingthe equivalent of 75 g anhydrous glucose dissolved in water.Impaired fasting glucose (IFG) is defined as FPG of 100–125 mg/dL. Impaired glucose tolerance (IGT) is a2-hour plasma glucose of 140–199 mg/dL after a 75 g OGTT. Although the OGTT is more sensitive and modestlymore specific than FPG to diagnose diabetes, it is poorly reproducible and rarely performed in practice. Becauseof ease of use, acceptability to patients, and lower cost, the FPG is the preferred diagnostic test. It should be notedthat the vast majority of people who meet diagnostic criteria for diabetes by OGTT, but not by FPG, will havean A1C value 7.0%. The use of the A1C for the diagnosis of diabetes is not recommended by the ADA at thistime. Pre-diabetes is defined as having either IFG or IGT.Bibliography1. <strong>American</strong> <strong>Diabetes</strong> <strong>Association</strong>. Standards of medical care in diabetes – 2007. <strong>Diabetes</strong> Care 2007;30(Supp1).Item 7Answer EAccording to the ADA guidelines:1. Testing for diabetes should be considered in all individuals at age 45 years and above, particularly in thosewith a BMI 25 kg/m 2 and, if normal, should be repeated at 3-year intervals.2. Testing should be considered at a younger age or be carried out more frequently in individuals who areoverweight (BMI 25 kg/m 2 *) and have additional risk factors: Are habitually physically inactive. Have a first-degree relative with diabetes. Are members of a high-risk ethnic population (e.g., African-<strong>American</strong>, Latino, Native <strong>American</strong>,Asian <strong>American</strong>, Pacific Islander). Have delivered a baby weighing 9 lb or have been diagnosed with GDM. Are hypertensive (blood pressure, 140/90 mm Hg). Have an HDL cholesterol level 35 mg/dL (0.90 mmol/L) and/or a triglyceride level 250 mg/dL(2.82 mmol/L). Have polycystic ovary syndrome (PCOS). On previous testing, had IGT or IFG. Have other clinical conditions associated with insulin resistance (e.g., PCOS or acanthosis nigricans). Have a history of vascular disease.* May not be correct for all ethnic groups.34<strong>American</strong> <strong>Diabetes</strong> <strong>Association</strong> <strong>Self</strong>-<strong>Assessment</strong> <strong>Program</strong>Educational (Learning) Critiques


This 35-year-old African-<strong>American</strong> man has at least two CHD risk factors (positive family history of CVD andhypertension) and, therefore, according to the NCEP ATP III guidelines, his LDL goal is 130 mg/dL. JNC7also recommends drug therapy for a blood pressure of 130/85 mm Hg in patients with multiple risk factors.The ADA does not recommend the routine use of OGTT to diagnose type 2 diabetes but does recommend thetesting of his overweight children for type 2 diabetes. The following is the criteria for testing for type 2 diabetesin children: Overweight (BMI 85th percentile for age and sex, weight for height 85th percentile, or weight 120%of ideal for height).Plus any two of the following risk factors: Family history of type 2 diabetes in first- or second-degree relative. Race/ethnicity (Native <strong>American</strong>, African-<strong>American</strong>, Latino, Asian <strong>American</strong>, Pacific Islander). Signs of insulin resistance or conditions associated with insulin resistance (acanthosis nigricans, hypertension,dyslipidemia, or PCOS). Maternal history of diabetes or GDM.Age of initiation: age 10 years or at onset of puberty, if puberty occurs at a younger age.Frequency: every 2 years.Test: FPG preferred.Clinical judgment should be used to test for type 2 diabetes in high-risk patients who do not meet these criteria.Bibliography1. <strong>American</strong> <strong>Diabetes</strong> <strong>Association</strong>. Standards of medical care in diabetes – 2007. <strong>Diabetes</strong> Care 2007;30(Supp1).Item 8Answer CThe ADA states that a graded exercise test with electrocardiogram (ECG) monitoring should be seriouslyconsidered before undertaking aerobic physical activity with intensity exceeding the demands of everyday living(more intense than brisk walking) in previously sedentary patients with diabetes whose 10-year risk of a coronaryevent is likely to be 10%. A diet prescription that focuses on the foods the patient is used to eating should alsobe implemented.This patient’s 39-year-old sister does not have a 10-year risk of coronary event 10% (her risk is 5%) and,therefore, a stress test is not recommended. In regard to treatment, the ADA recommends early intervention withmetformin in combination with lifestyle changes (Figure 1). The ADA does not recommend low-carbohydratediets (restricting to total carbohydrate (130 g/day). The long-term effects of these diets are unknown, andalthough such diets produce short-term weight loss, maintenance of weight loss is similar to that from low-fat dietsand the impact on CVD risk profile is uncertain. Monitoring carbohydrate, whether by carbohydrate counting,exchanges, or experience-based estimation, remains a key strategy in achieving glycemic control. For individualswith diabetes, the use of glycemic index and glycemic load may provide a modest additional benefit for glycemiccontrol over that observed when total carbohydrate is considered alone. Therefore, there is not sufficient evidenceto recommend use of glycemic index or glycemic load for prevention of diabetes, although foods high in fiber areencouraged.Educational (Learning) Critiques<strong>American</strong> <strong>Diabetes</strong> <strong>Association</strong> <strong>Self</strong>-<strong>Assessment</strong> <strong>Program</strong>35


DiagnosisLifestyle Intervention + MetforminNo A1C 7%Yes *Add Basal Insulin(Most effective)Add Sulfonylurea(Least expensive)Add Glitazone(No hypoglycemia)No A1C 7% Yes * No A1C 7% Yes * No A1C 7% Yes *Intensify Insulin Add Glitazone † Add Basal Insulin Add Sulfonylurea †No A1C 7%Yes *No A1C 7%Yes *Add Basal or Intensify InsulinIntensive Insulin + Metformin +/- GlitazoneFigure 1. Algorithm for the metabolic management of type 2 diabetes. Reinforce lifestyle intervention at every visit.* Check A1C every 3 months until 7% and then at least every 6 months.† Although three oral agents can be used, initiation and intensification of insulin therapy is preferred based on effectiveness and expense.Sulfonylurea and glitazone are recommended as add-on therapy to metformin if the A1C is 7.0%. Even thoughthis patient has low HDL and glitazone has diabetic dyslipidemic benefits and sulfonylurea lowers glucose rapidly,the primary initial therapy should be metformin due to cost considerations, the absence of hypoglycemia, andevidence of benefit.Bibliography1. <strong>American</strong> <strong>Diabetes</strong> <strong>Association</strong>. Standards of medical care in diabetes – 2007. <strong>Diabetes</strong> Care 2007;30(Supp1).Item 9Answer AAccording to the ADA guidelines: Patients with a systolic blood pressure of 130–139 mm Hg or a diastolic blood pressure of 80–89 mm Hgshould be given lifestyle and behavioral therapy alone for a maximum of 3 months and then, if targets are notachieved, in addition, be treated with pharmacological agents that block the rennin-angiotensin system. Initial drug therapy for those with a blood pressure 140/90 mm Hg should be with a drug class demonstratedto reduce CVD events in patients with diabetes (ACE inhibitors, angiotensin receptor blockers [ARBs]‚β-blockers, diuretics, and calcium channel blockers). All patients with diabetes and hypertension should be treated with a regimen that includes either an ACEinhibitor or an ARB. If one class is not tolerated, the other should be substituted. If needed to achieve bloodpressure targets, a thiazide diuretic should be added. In individuals without overt CVD: The primary goal is an LDL 100 mg/dL (2.6 mmol/L). For those over the age of 40 years, statin therapy to achieve an LDL reduction of 30–40% regardlessof baseline LDL levels is recommended. For those under the age of 40 years but at increased risk due to other cardiovascular risk factors who do notachieve lipid goals with lifestyle modifications alone, the addition of pharmacological therapy is appropriate. Other goals include: Lower triglycerides to 150 mg/dL (1.7 mmol/L) and raise HDL cholesterol to 40 mg/dL (1.0 mmol/L).In women, an HDL goal 10 mg/dL higher (50 g/dL) should be considered.36<strong>American</strong> <strong>Diabetes</strong> <strong>Association</strong> <strong>Self</strong>-<strong>Assessment</strong> <strong>Program</strong>Educational (Learning) Critiques


The primary treatment for dyslipidemia is a statin, unless the triglycerides are 500 mg/dL.Bibliography1. <strong>American</strong> <strong>Diabetes</strong> <strong>Association</strong>. Standards of medical care in diabetes – 2007. <strong>Diabetes</strong> Care 2007;30(Supp1).Item 10Answer EThe ADA recommends all of the following ancillary tests as part of the initial workup of patients with newlydiagnosedtype 2 diabetes. Urinary evaluation for microalbuminuria is important to assess early renal damage andpotentially increase the aggressiveness of blood pressure control. Patients with type 2 diabetes should also undergoa dilated comprehensive eye exam by an ophthalmologist or optometrist as well as screening for distal symmetricpolyneuropathy using simple clinical tests (i.e., assess vibratory response in both feet). A comprehensive foot examis needed to evaluate for potential ulcers or loss of sensation with the appropriate referral for special footwear orpodiatric care.Bibliography1. <strong>American</strong> <strong>Diabetes</strong> <strong>Association</strong>. Standards of medical care in diabetes – 2007. <strong>Diabetes</strong> Care 2007;30(Supp1):S4–S41.2. Nathan DM, et al: Management of hyperglycemia in type 2 diabetes: A consensus algorithm for the initiation and adjustment of therapy:a consensus statement from the <strong>American</strong> <strong>Diabetes</strong> <strong>Association</strong> and the European <strong>Association</strong> for the Study of <strong>Diabetes</strong>. <strong>Diabetes</strong> Care2006;29:1963–1972.Item 11Answer AThe ADA website has a risk calculator that can be a very valuable educational tool to assist clinicians to assess riskof diabetes. The website link is www.diabetes.org/diabetesphd. This website is very patient-friendly and can helpeducate regarding the benefits of weight reduction and other factors to reduce the risk of diabetes, heart disease,stroke, and kidney disease. The <strong>Diabetes</strong> PhD risk calculator can also powerfully demonstrate the benefits of evenmodest weight loss in reducing the risk of type 2 diabetes. In this patient, a 10% weight loss (20 lbs) would reduceher risk of developing type 2 diabetes to less than 10%. Weight loss is the most powerful modifier for the preventionof type 2 diabetes.In the <strong>Diabetes</strong> Prevention <strong>Program</strong> (DPP), the 3234 enrolled subjects were slightly younger (mean age 51 years)and more obese (mean BMI 34 kg/m 2 ) but had nearly identical glucose intolerance compared with subjects inthe Finnish study. About 45% of the participants were from minority groups (e.g., African-<strong>American</strong>, Hispanic),and 20% were 60 years of age. Subjects were randomized to one of three intervention groups, which includedthe intensive nutrition and exercise counseling (“lifestyle”) group or either of two masked medication treatmentgroups: the biguanide metformin group or the placebo group. The latter interventions were combined withstandard diet and exercise recommendations. After an average follow-up of 2.8 years (range 1.8–4.6 years), a 58%relative reduction in the progression to diabetes was observed in the lifestyle group (absolute incidence 4.8%), anda 31% relative reduction in the progression of diabetes was observed in the metformin group (absolute incidence7.8%) compared with control subjects (absolute incidence 11.0%). On average, 50% of the lifestyle group achievedthe goal of 7% weight reduction, and 74% maintained at least 150 min/week of moderately intense activity.No serious side-effects were seen in any group.Bibliography1. <strong>Diabetes</strong> Prevention Research Group: Reduction in the evidence of type 2 diabetes with life-style intervention or metformin.N Engl J Med 2002;346:393–403.2. The <strong>Diabetes</strong> Prevention <strong>Program</strong> Research Group: The <strong>Diabetes</strong> Prevention <strong>Program</strong>: baseline characteristics of the randomizedcohort. <strong>Diabetes</strong> Care 2000;23:1619–1629.Educational (Learning) Critiques<strong>American</strong> <strong>Diabetes</strong> <strong>Association</strong> <strong>Self</strong>-<strong>Assessment</strong> <strong>Program</strong>37


Item 12Answer DThe ADA recently published a consensus statement of the implications for care in patients with pre-diabetes(either impaired fasting glucose and/or impaired glucose tolerance). The ADA Panel recommended, based onclinical trial evidence and cost considerations, that only metformin be considered as drug therapy for individualswith IFG/IGT. In the DPP study, the subsets of the study cohort that had substantially increased benefit frommetformin were those participants 60 years of age and those who had a BMI 35 kg/m 2 . Therefore, the Panelalso recommends that metformin be limited to such individuals. Since individuals with associated risk factors fordiabetes, e.g., family history in first-degree relatives, elevated triglycerides, low HDL cholesterol, and hypertension,are more likely to progress to diabetes, the presence of one or more of these factors may contribute to the decisionto treat with metformin. In addition, to better target a population likely to benefit from metformin therapy, anunpublished analysis of data from the DPP study suggests that an A1C 6.0% approximately doubles the rateof progression to diabetes in an IFG/IGT population.A summary of these recommendations in shown in Table 1. Future recommendations may include othermedications if they prove to be effective, have a good safety profile, are tolerable, and are of relatively low cost.Table 1. Treatment recommendation for individuals with IFG, IGT, or both.PopulationIFG or IGTIndividuals with IFG and IGT and any of the following: 60 years of age. BMI 35 kg/m 2 . Family history of diabetes in first-degree relatives. Elevated triglycerides. Reduced HDL-C. Hypertension. A1C 6.0%.TreatmentLifestyle modification (i.e., 5–10% weight loss and moderateintensity physical activity 30 min/day)Lifestyle modification (as above) and/or metformin** Metformin 850 mg b.i.d.Bibliography1. Nathan DM, et al: Impaired fasting glucose and impaired glucose tolerance. <strong>Diabetes</strong> Care 2007;30:753–759.Item 13Answer BThe ADA’s treatment algorithm for hyperglycemia strongly encourages the initiation of metformin in conjunctionwith lifestyle changes. Therefore, although this patient has made some progress with weight loss, metformintreatment should be initiated. Sulfonylureas are second-line agents and can effectively lower blood glucose acutelyin symptomatic individuals, but this patient’s hyperglycemia is not severe, and she is asymptomatic. Althoughshe has dyslipidemia, the ADA recommends TZDs as second-line agents but should be considered in patientswith low HDL/elevated triglycerides. The hyperglycemic management guidelines were released prior to theapproval of sitagliptin, which is a category B drug for use in women of child-bearing potential, but metforminis still the first-line agent.Bibliography1. Nathan DM, et al: Management of hyperglycemia in type 2 diabetes: A consensus algorithm for the initiation and adjustment of therapy:a consensus statement from the <strong>American</strong> <strong>Diabetes</strong> <strong>Association</strong> and the European <strong>Association</strong> for the Study of <strong>Diabetes</strong>. <strong>Diabetes</strong> Care2006;29:1963–1972.38<strong>American</strong> <strong>Diabetes</strong> <strong>Association</strong> <strong>Self</strong>-<strong>Assessment</strong> <strong>Program</strong>Educational (Learning) Critiques


Items 14–18Answers 14 (D); 15 (B); 16 (A); 17 (E); 18 (C)Thiazolidinediones are contraindicated in patients with NYHA Class III/IV cardiac failure because of concernsthat they may precipitate pulmonary edema, but there is no evidence that they are harmful in Class I heart failure.Rosiglitazone has recently been implicated as increasing the incidence of myocardial infarction and a borderlinesignificant increase in death in a meta-analysis. Thiazolidinediones also cause a modest reduction in the hematocritprobably by a hemdilution effect. Gemfibrozil, but not fenofibrate, inhibits glucuronidation of statins. This is apathway that had not been recognized earlier as being important in statin catabolism. As a consequence, statinlevels have been demonstrated to be increased in subjects receiving gemfibrozil. Fenofibrate is extensively metabolizedin the kidney, and therefore should be dose-adjusted in patients with moderate-to-severe renal impairment.Gemfibrozil requires dose adjustment only in patients with severe renal impairment. In the clinical scenariodescribed in Item 17 the thiazolidinediones and fibrate are not contraindicated since the liver enzymes are less thantwice elevated, and these drugs may improve fatty liver changes.Bibliography1. Nesto RW, et al: <strong>American</strong> Heart <strong>Association</strong>; <strong>American</strong> <strong>Diabetes</strong> <strong>Association</strong>. Thiazolidinedione use, fluid retention, and congestiveheart failure: a consensus statement from the <strong>American</strong> Heart <strong>Association</strong> and <strong>American</strong> <strong>Diabetes</strong> <strong>Association</strong>. Circulation2003;108(23):2941–8.2. Lebovitz HE. Differentiating members of the thiazolidinedione class: a focus on safety. <strong>Diabetes</strong> Metab Res Rev 2002;18(Suppl 2):S23–9.3. Prueksaritanont T, et al: Effects of fibrates on metabolism of statins in human hepatocytes. Drug Metab Dispos 2002;30(11):1280–7.4. Gajdos M, et al: Fibrates and renal function. Clin Nephrol 2003;60(1):65–6.5. On-Line: N Engl J Med 5/20/07.Items 19–23Answers 19 (A); 20 (A); 21 (C); 22 (D); 23 (E)The importance of adipocytokines to the genesis of insulin resistance and atherosclerosis is assuming increasingimportance. Among these, adiponectin has been shown to both enhance insulin sensitivity and inhibit monocyteadhesion and infiltration of the vascular wall. By contrast, free fatty acids cause insulin resistance in part by interferingwith hepatic insulin degradation causing hyperinsulinemia which leads to down-regulation of insulinreceptors and insulin resistance. TNF-α increases insulin resistance by interfering with insulin-signaling pathways.PAI-1 localizes in plaque where it may promote coagulation.Bibliography1. Molecular mechanisms of insulin resistance and the role of the adipocyte: Int J Obes Relat Metab Disord 2000;24(Suppl)4:S23–7.2. Hoffstedt J, et al: Adipose tissue adiponectin production and adiponectin serum concentration in human obesity and insulinresistance.J Clin Endocrinol Metab 2004;89(3):1391–6.3. Hennes MM, et al: Effects of free fatty acids and glucose on splanchnic insulin dynamics. <strong>Diabetes</strong> 1997;46(1):57–62.Item 24Answer BThe biguanides, represented by metformin in the United States, are effective oral hypoglycemic agents. They arealso effective when added to other oral hypoglycemic agents, such as thiazolidinediones or sulfonylureas, or whenadded to insulin. They are usually dosed with meals. Use of biguanides has been associated with lactic acidosisrarely with no obvious cause (50% death rate if it does occur) and especially in patients with renal insufficiency,significant liver disease, or compromised LV function (90% death rates).Bibliography1. Mudaliar S, et al: The oral antidiabetic agents. In: Ellenberg and Rifkin’s <strong>Diabetes</strong> Mellitus, 6th Ed. Porte D, et al (eds).McGraw-Hill, 2003;531–564.Educational (Learning) Critiques<strong>American</strong> <strong>Diabetes</strong> <strong>Association</strong> <strong>Self</strong>-<strong>Assessment</strong> <strong>Program</strong>39


Item 25Answer BInsulin resistance is associated with increased risk for cardiovascular disease. There have been multiple mechanismssuggested for this association. However, recently there has been increasing support for the pro-inflammatorystate that accompanies insulin resistance as being pathophysiologically important for accelerated large vesseldisease. Patients with insulin resistance sometimes have elevated homocysteine levels. However, the relationshipbetween homocysteine and the accelerated vascular disease of insulin resistance is less strong (therapy with folicacid decreases homocysteine levels but does not reduce CV events). Patients with insulin resistance have increasedplasminogen activator inhibitor levels and, therefore, a pro-thrombotic state. While patients with insulin resistancealso have elevated insulin levels, the association between insulin levels and vascular disease is most likely relatedto insulin resistance and not to direct toxic effect of insulin on the vasculature.Bibliography1. Haffner S, et al: Insulin-resistant prediabetic subjects have more atherogenic risk factors than insulin-sensitive prediabetic subjects.Implications for preventing coronary heart disease during the prediabetic state. Circulation 2000;101:975–980.2. Hunt K, et al: Elevated carotid artery intima-media thickness levels in individuals who subsequently develop Type 2 diabetes.Arterioscler Thromb Vasc Biol 2003;23:1845–1850.3. Soinio M, et al: Elevated plasma homocysteine level is an independent predictor of coronary heart disease events in patients withType 2 diabetes mellitus. Ann Intern Med 2004;140:94–100.Item 26Answer DThe ability to monitor blood glucose at home has revolutionized management of patients with diabetes. Bloodglucose levels are of value to physicians, and can be obtained either several times per week or several times per day,depending on the clinical circumstance. While patients on insulin require more frequent monitoring patients withtype 2 diabetes on oral hypoglycemic therapy will also benefit from home monitoring. For patients on insulintherapy, measuring the fasting blood glucose is the most reliable indicator for adjustment of the basal insulin dose.In the most intensively controlled patients, it is useful to measure blood glucose before meals and at bedtime.Bibliography1. <strong>American</strong> <strong>Diabetes</strong> <strong>Association</strong>: Clinical Practice Recommendations 2004. Standards of medical care in diabetes.<strong>Diabetes</strong> Care 27(Suppl. 1):2004;S15–S35.40<strong>American</strong> <strong>Diabetes</strong> <strong>Association</strong> <strong>Self</strong>-<strong>Assessment</strong> <strong>Program</strong>Educational (Learning) Critiques


Items 27–31Answers 27 (A); 28 (D); 29 (C); 30 (B); 31 (E)There are now several classes of oral hypoglycemic agents available. Because these major classes have complementarymechanisms of action, they are often useful as combination therapy in individual patients. Sulfonylureaswork primarily by increasing insulin secretion from β-cells. Thiazolidinediones are insulin sensitizers that increaseglucose disposal by increasing its uptake into muscle. The biguanides have a favorable effect on fasting blood glucoseby suppressing glucose production in the liver. The alpha-glucosidase inhibitors are not as efficacious for reducingoverall glycemia as sulfonylureas, thiazolidinediones or biguanides; however, they are useful for reducing postprandialblood glucose spikes, as they delay glucose uptake in the intestine and, thereby, inhibit glucose absorption.Ingestion of foodGI tractSitagliptin(DPP-4 inhibitor)Release ofactive incretinsGLP-1 and GIPInactiveGLP-1DPP-4enzymeInactiveGIPGlucosedependentInsulin(GLP-1and GIP)PancreasBeta cellsAlpha cellsGlucosedependentGlucagon(GLP-1)Glucoseuptake byperipheraltissueHepaticglucoseproductionBlood glucosein fasting andpostprandial statesFigure 2. Mechanism of action of sitagliptin.The incretin hormones GLP-I and GIP are released by the intestine throughout the day, and levels are increased inresponse to a meal. The incretins are part of an endogenous system involved in the physiologic regulation of glucosehomeostasis.When blood glucose concentrations are normal or elevated, GLP-I and GIP increase insulin synthesis and releasefrom pancreatic β-cells by intracellular signaling pathways involving cyclic AMP. With higher insulin levels, tissueglucose uptake is enhanced.In addition, GLP-I lowers glucagon secretion from pancreatic α-cells. Decreased glucagon levels, along with higher insulin levels, lead to reduced hepatic glucose productionand are associated with a decrease in blood glucose levels in the fasting and post-prandial states. The beta-cell effects of GLP-I and GIP are glucose-dependent, but only the effect of GLP-1 on the alpha cellis glucose-dependent. The glucose-dependency effect of GLP-1 on alpha- and beta-cells translate to a benefitof rare hypoglycemia with incretin mimetics.The activity of GLP-I and GIP is limited by the DPP-4 enzyme, which rapidly inactivates incretin hormones.Concentrations of the active intact hormones are increased by sitagliptin, thereby increasing and prolonging theaction of these hormones. (Figure 2).Bibliography1. Mudaliar S, et al: The oral antidiabetic agents. In: Ellenberg and Rifkin’s <strong>Diabetes</strong> Mellitus, 6th Ed. Porte D, et al (eds). McGraw-Hill,2003;531–564.Educational (Learning) Critiques<strong>American</strong> <strong>Diabetes</strong> <strong>Association</strong> <strong>Self</strong>-<strong>Assessment</strong> <strong>Program</strong>41


Items 32–35Answers 32 (B); 33 (C); 34 (A); 35 (D)The oral hypoglycemic agents are generally safe; however, each major class is associated with specific serious sideeffects.Sulfonylureas, because they increase insulin secretion, are the class of oral hypoglycemic agents most closelyrelated to production of symptomatic hypoglycemia. The thiazolidinediones may increase fluid retention andworsen peripheral edema. Their use in patients with symptomatic congestive heart failure is, therefore, not recommended.Biguanides most often give rise to gastrointestinal side-effects including mild nausea and some diarrhea,but this can be minimized by titrating to a maximal effective dose of 2 g/day over 2 weeks. These side-effects aregenerally self-limited. However, in patients with renal insufficiency or impaired cardiac function, biguanides havebeen rarely associated with lactic acidosis (see Critique for Item 24). Alpha-glucosidase inhibitors decrease thedigestion of carbohydrates which are converted into gas and can cause bloating.Bibliography1. Mudaliar S, et al: The oral antidiabetic agents. In: Ellenberg and Rifkin’s <strong>Diabetes</strong> Mellitus, 6th Ed. Porte D, et al (eds).McGraw-Hill, 2003;531–564.Item 36Answer DAll of the options in this item have been shown to be beneficial as each can lead to weight loss. A reduction inweight of as little as 4 kg has been shown to improve hyperglycemia. Whereas dietary and physical activity adjustmentshave been universally recommended and remarkably useful in some patients, weight loss following bariatricsurgery has been shown to be consistently effective in virtually eliminating diabetes if a mean weight loss of 20 kgor more can be sustained. Although not appropriate for all patients, in properly-selected obese patients, bariatricsurgery can be an effective option. The results of recent studies have shown that bariatric surgery decreasedmortality rate in selected obese patients with type 2 diabetes. Studies of weight loss medications, such as orlistatwhich reduces fat absorption and sibutramine which causes a modest decrease in appetite have demonstrated highdrop-out rates, significant side-effects, and cannot be recommended at this time for the treatment of diabetes.Bibliography1. Sjostrom L, et al: Swedish Obese Subjects Study Scientific Group. Lifestyle, diabetes, and cardiovascular risk factors 10 years afterbariatric surgery. N Engl J Med 2004;351:2683–2693.2. Nathan DM, et al: Management of hyperglycemia in type 2 diabetes: A consensus statement from the <strong>American</strong> <strong>Diabetes</strong> <strong>Association</strong>and the European <strong>Association</strong> for the Study of <strong>Diabetes</strong>. <strong>Diabetes</strong> Care 2006;29:1963–1972.Item 37Answer EExercise and weight loss have a beneficial effect on diabetes control but also on associated cardiovascularrisk factors, such as blood pressure and lipids. There is a psychological boost as well in exercising regularlyand seeing this improve your glycemic control and other risk factors. While lower weight and improved diabeticcontrol may slow the progression of neuropathy in the long-term, in the short-term the newly-exercising patientwith neuropathy is often at increased risk of foot trauma or ulceration. One should caution all patients withdiabetes, especially those with neuropathy, to look carefully at their feet everyday, particularly after exercise.It is also important to have properly-fitting shoes and other equipment to lessen the possibility of injury.Bibliography1. Nathan DM, et al: Management of hyperglycemia in type 2 diabetes: A consensus statement from the <strong>American</strong> <strong>Diabetes</strong> <strong>Association</strong>and the European <strong>Association</strong> for the Study of <strong>Diabetes</strong>. <strong>Diabetes</strong> Care 2006;29:1963–1972.2. <strong>Diabetes</strong> Prevention <strong>Program</strong> Research Group: Impact of intensive lifestyle and metformin therapy on cardiovascular disease risk factorsin the <strong>Diabetes</strong> Prevention <strong>Program</strong>. <strong>Diabetes</strong> Care 2005;28:888–894.42<strong>American</strong> <strong>Diabetes</strong> <strong>Association</strong> <strong>Self</strong>-<strong>Assessment</strong> <strong>Program</strong>Educational (Learning) Critiques


Item 38Answer CMetformin is recommended as first line of treatment in most patients with newly-diagnosed diabetes that requiremedication for all of the reasons mentioned in this item, except for option (C). While metformin is generallywell-tolerated it may have gastrointestinal side-effects, such as diarrhea or nausea, especially at higher doses.More worrisome is the potentially fatal possibility of lactic acidosis which is actually very rare (1 case per 100,000treated patients). The risk of lactic acidosis can be reduced by avoiding metformin in patients with renal insufficiency(creatinine 1.4 mg/dL in women, 1.5 mg/dL in men), in association with the use of iodinated contrastor with any condition that predisposes the patient to hypotension or hypoxia.Bibliography1. Nathan DM, et al: Management of hyperglycemia in type 2 diabetes: A consensus statement from the <strong>American</strong> <strong>Diabetes</strong> <strong>Association</strong>and the European <strong>Association</strong> for the Study of <strong>Diabetes</strong>. <strong>Diabetes</strong> Care 2006;29:1963–1972.2. Salpeter S, et al: Risk of fatal and nonfatal lactic acidosis with metformin use in type 2 diabetes mellitus. Cochrane Database Syst Rev2006;CD002967.Item 39Answer CThe sulfonylurea medications increase insulin secretion which lowers blood glucose levels. This can causeincreased hunger, hypoglycemia and weight gain. It is common for a patient to gain about 2 kg when startedon a sulfonylurea, so they must be used with caution in those that are already overweight. Caution should also beadvised in the elderly that are more prone to hypoglycemia and drug interactions. Urinary tract infections (UTIs)are common in patients with diabetes and sulfa drugs are commonly prescribed for UTIs. However, in one studyof over 900 elderly patients admitted to the hospital for hypoglycemia related to a taking a sulfonylurea, comparedwith matched controls the patients with hypoglycemia were more than 6 times as likely to have been treated witha sulfa antibiotic in the previous week. There is no logic to prescribe a sulfonylurea and glinide together as bothact on the same receptor on the beta-cell. Like almost any medication, the sulfonylureas can cause a rash.However, a clear consistent adverse effect on cardiovascular risk has not been established. While the UniversityGroup <strong>Diabetes</strong> <strong>Program</strong> suggested an increase in mortality when sulfonylureas are used in patients with type 2diabetes, this was not confirmed by the UK Prospective <strong>Diabetes</strong> Study.Bibliography1. Juurlink DN, et al: Drug-drug interactions among elderly patients hospitalized for drug toxicity. JAMA 2003 Apr 2;289(13):1652–8.PMID: 12672733.2. Klimt CR, et al: The University Group <strong>Diabetes</strong> <strong>Program</strong>: a study of the effect of hypoglycemic agents on vascularcomplications in patients with adult-onset diabetes. I. Design, methods and baseline characteristics. II. Mortality results.<strong>Diabetes</strong> 1970;19(Suppl. 2):747–830.3. UK Prospective <strong>Diabetes</strong> Study (UKPDS) Group: Intensive blood glucose control with sulphonylureas or insulin comparedwith conventional treatment and risk of complication in patients with type 2 diabetes (UKPDS 33). Lancet 1998;352:837–853.Educational (Learning) Critiques<strong>American</strong> <strong>Diabetes</strong> <strong>Association</strong> <strong>Self</strong>-<strong>Assessment</strong> <strong>Program</strong>43


Item 40Answer CThe only significant drug-drug interaction with the glinides (e.g., repaglinide, nateglinide) concerns gemfibrozil,which can lead to hypoglycemia by reducing the metabolism of either glinide drug by the liver. Therefore, whenhypertriglyceridemia and diabetes coexist, which often occurs, caution must be exercised in choosing therapy.A method other than gemfibrozil, such as control of diabetes, dietary measures, fenofibrate or omega-3 fatty acidsshould be used if triglyceride-lowering is needed.Bibliography1. Hasslacher C. Multinational Repaglinide Renal Study Group. Safety and efficacy of repaglinide in type 2 diabetic patients with andwithout impaired renal function. <strong>Diabetes</strong> Care 2003 Mar;26(3):886–91.2. Blickle JF. Meglitinide analogues: a review of clinical data focused on recent trials. <strong>Diabetes</strong> Metab 2006 Apr;32(2):113–20.Item 41Answer AMetformin is associated with an expected decrease in A1C level of 1.5%. The only biguanide presently marketedin the United States, metformin has been called an “insulin sensitizer” because it lowers blood glucose levelswithout directly increasing insulin production or secretion by the pancreas. Its main mechanism however isto reduce gluconeogenesis in the liver, thereby reducing glucose production. It likely exerts this effect throughalterations in AMP-kinase. Reductions in insulin resistance in the peripheral tissues have been observed but theresults are inconsistent.Bibliography1. Inzucchi SE, et al: Efficacy and metabolic effects of metformin and troglitazone in type II diabetes mellitus. N Engl J Med1998;338:867–872.2. Hundal RS, et al: Mechanism by which metformin reduces glucose production in type 2 diabetes. <strong>Diabetes</strong> 2000;49:2063–2069.3. Yu JG, et al: A comparison of troglitazone and metformin on insulin requirements in euglycemic intensively insulin-treated type 2diabetic patients. <strong>Diabetes</strong> 1999;48:2414–242.Item 42Answer DGlitazones are associated with an expected decrease in A1C level of 0.5–1.4%. Pioglitazone and rosiglitazone arethe two thiazolidinediones (TZDs) that are currently available. The primary mechanism of action of the TZDs is toenhance insulin sensitivity by improving the utilization of glucose by the peripheral adipocytes and skeletal muscle.TZDs create this effect by stimulating a nuclear transcription factor called PPAR-γ. These receptors impact lipidand carbohydrate metabolism by activating many (1000), not just several, genes in various tissues. PPAR-γ ismost highly expressed in adipocytes and TZDs and are thought to act within fat cells to effect the production offree fatty acids, adiponectin and TNF-α, all of which impact insulin sensitivity. Some have described a reduction inhepatic glucose production but not to the extent of metformin.Bibliography1. Frias JP, et al: Metabolic effects of troglitazone therapy in type 2 diabetic, obese, and lean normal subjects. <strong>Diabetes</strong> Care 2000;23: 64–69.2. Nolan JJ, et al: Improvement in glucose tolerance and insulin resistance in obese subjects treated with troglitazone. N Engl J Med1994;331:1188–1193.Item 43Answer CAlpha-glucosidase inhibitors (AGIs) are associated with an expected decrease in A1C level of 0.5–0.8%.This drug class includes acarbose and miglitol. AGIs work by delaying the absorption of carbohydrates in thegut. This has the most significant effect on the post-prandial blood glucose level so these medications are takenwith meals. The AGIs directly inhibit an enzyme that breaks down complex carbohydrates on the brush border ofthe enterocyte. Unfortunately, the mechanism that creates their efficacy also yields their most significant side-effect(intestinal bloating and gas). This is because the carbohydrates that are not absorbed are acted upon by the floraof the gut to produce gas.44<strong>American</strong> <strong>Diabetes</strong> <strong>Association</strong> <strong>Self</strong>-<strong>Assessment</strong> <strong>Program</strong>Educational (Learning) Critiques


Bibliography1. Lebowitz HE. α-glucosidase inhibitors as agents in the treatment of diabetes. <strong>Diabetes</strong> Rev 1998;6:132–145.2. Hanefeld M, et al: Therapeutic potentials of acarbose as first-line drug in NIDDM insufficiently treated with diet alone. <strong>Diabetes</strong> Care1991;14 :732–737.3. Hotta N, et al: Long-term effect of acarbose on glycaemic control in non-insulin-dependent diabetes mellitus: a placebo-controlleddouble-blind study. Diabet Med 1993;10:134–138.Item 44Answer BGlinides (also known as meglitinides) are associated with an expected decrease in A1C level of 1–1.5%. Twoglinides are available in the US at this time. Repaglinide is somewhat more efficacious, reducing A1C levels by1.5%. Nateglinide when used as monotherapy reduces A1C by about 1%. Each of these works by acting upon thesame receptor as the sulfonylureas although at a different site. This causes a sequence of events that includesclosure of the ATP-dependent K channels which stimulate insulin secretion from the pancreas. These medicationsare taken just prior to meals and act primarily on the post-prandial glucose level.Bibliography1. Scarsi M,et.al: Sulfonylureas and Glinides Exhibit PPAR-γ Activity: A Combined Virtual Screening and Biological Assay Approach.Mol Pharmacol 2006 Nov 3.2. Nathan DM, et al: Management of hyperglycemia in type 2 diabetes: A consensus statement from the <strong>American</strong> <strong>Diabetes</strong> <strong>Association</strong>and the European <strong>Association</strong> for the Study of <strong>Diabetes</strong>. <strong>Diabetes</strong> Care 2006;29:1963–1972.Item 45Answer ESitagliptin is associated with an expected decrease in A1C level of 0.5–1.5%. Sitagliptin is a dipeptidyl peptidase-4(DPP-IV) inhibitor. This acts upon the incretin system, which is a group of hormones active in the gut. One ofthese hormones is GLP-I. DPP-IV inhibitors reduce the breakdown of many peptides, including GLP-I, therebyincreasing their activity. The main effect is to improve glucose-dependent insulin secretion and a glucosedependentsuppression of inappropriate glucagon secretion. In addition, it results in an increased endogenousGLP-1 effect of 2–4 times normal, which does not sufficiently suppress appetite or slow gastric emptying enoughto decrease weight. Though it is weight-neutral, it has no increased GI side-effects compared to placebo.Bibliography1. Nathan DM, et al: Management of hyperglycemia in type 2 diabetes: A consensus statement from the <strong>American</strong> <strong>Diabetes</strong> <strong>Association</strong>and the European <strong>Association</strong> for the Study of <strong>Diabetes</strong>. <strong>Diabetes</strong> Care 2006;29:1963–1972.2. Aschner P, et al: Effect of the dipeptidyl peptidase-4 inhibitor sitagliptin as monotherapy on glycemic control in patients with type 2diabetes. <strong>Diabetes</strong> Care 2006 Dec;29(12):2632–7.Item 46Answer CThe only antihypertensive agents proven to prevent progression to microalbuminuria are the angiotensinconvertingenzyme inhibitors (ACE-I). A recent review looked at the question of preventing nephropathy inpatients with type 1 or type 2 diabetes who had normal kidney function with or without hypertension. The absenceof nephropathy was defined as albumin excretion less than 30 mg/day on three serial measurements. Most of thetrials reviewed involved patients with type 2 diabetes with hypertension.ACE-Is were the only agents that clearly showed evidence of preventing progression to microalbuminuria comparedto placebo or calcium channel blockers. The number of patients with diabetes one needed to treat to preventprogression of one patient to microalbuminuria was 25. ACE-Is did not reduce progression to end-stage renaldisease or all-cause mortality. There were not enough studies to determine if normotensive patients received anybenefit.Educational (Learning) Critiques<strong>American</strong> <strong>Diabetes</strong> <strong>Association</strong> <strong>Self</strong>-<strong>Assessment</strong> <strong>Program</strong>45


Angiotensin receptor blockers (ARBs) have also been shown to slow the progression of nephropathy associatedwith diabetes. The summary of a very recent review of the effect of ACE-Is and ARBs is worth quoting in full.“Fifty studies (13215 patients) were identified. Thirty eight compared ACE-I with placebo, five compared ARBswith placebo and seven compared ACE-Is and ARBs directly. There was no significant difference in the risk ofall-cause mortality for ACE-I versus placebo (RR 0.91, 95% CI 0.71–1.17) and ARBs versus placebo (RR 0.99,95% CI 0.85–1.17). A subgroup analysis of studies using full-dose ACE-Is versus studies using half or less thanhalf the maximum tolerable dose of ACE-Is showed a significant reduction in the risk of all-cause mortality withthe use of full-dose ACE-I (RR 0.78, 95% CI 0.61–0.98). Baseline mortality rates were similar in the ACE-I andARB studies. The effects of ACE-Is and ARBs on renal outcomes (ESKD, doubling of creatinine, prevention ofprogression of micro- to macroalbuminuria, remission of micro- to normoalbuminuria) were similarly beneficial.Reliable estimates of effect of ACE-Is versus ARBs could not be obtained from the three studies in which theywere compared directly because of their small sample size.”Bibliography1. Strippoli GFM, et al: Antihypertensive agents for preventing diabetic kidney disease. The Cochrane Database of Systematic Reviews2006 Issue 4.2. Strippoli GFM, et al: Angiotensin converting enzyme inhibitors and angiotensin II receptor antagonists for preventing the progressionof diabetic kidney disease. The Cochrane Database of Systematic Reviews 2006 Issue 4.Item 47Answer BWhile nearly all of the published studies regarding the use of statins and prevention of cardiovascular disease eventrates have shown a reduction, the Prospective Study of Pravastatin in the Elderly at Risk (PROSPER) trial founda trend toward harm regarding overall mortality. This trend did not reach the level of statistical significance.In the PROSPER study 5804 people aged 70–82 years were given pravastatin (40 mg/day; n = 2891) or placebo(n = 2913). Follow-up was 3.2 years on average and our primary endpoint was a composite of coronary death,non-fatal myocardial infarction, and fatal or non-fatal stroke. Pravastatin lowered LDL cholesterol concentrationsby 34% and reduced the incidence of the primary endpoints (hazard ratio 0.85, 95% CI 0.74–0.97, P = 0.014).Coronary heart disease death and non-fatal myocardial infarction risk were also reduced (0.81, 0.69–0.94,P = 0.006). Stroke risk was unaffected (1.03, 0.81–1.31, P = 0.8), but the hazard ratio for transient ischaemicattack was 0.75 (0.55–1.00, P = 0.051). New cancer diagnoses were more frequent on pravastatin than on placebo(1.25, 1.04–1.51, P = 0.020). However, a meta-analysis of five studies found no association between statin useand the risk of non-fatal and fatal cancers. All the other options in this item are correct.Bibliography1. Shepherd J, et al: Pravastatin in elderly individuals at risk of vascular disease (PROSPER): A randomised controlled trial.Lancet 2002;360:1623–30.Item 48Answer DAlpha-glucosidase inhibitors (AGIs), such as acarbose, work by delaying the intestinal absorption of glucose. Theyact primarily on post-prandial blood glucose levels and are unlikely to cause hypoglycemia. They generally have tobe dosed with each meal and the dose of acarbose is 50 mg t.i.d. The advantage of this class of drugs is they areunlikely to cause any systemic effects and are useful in patients with kidney or liver insufficiency where other drugsmay be contraindicated. Unfortunately, their use is limited by side-effects primarily gastrointestinal in that theundigested sugars are acted upon by intestinal bacteria to produce gas. Therefore patients with low tolerance forside-effects would not be good candidates for this class of medications.The AGIs are less potent than other classes of medications in reducing A1C from 0.5–0.8% so they are not the firstchoice for monotherapy if a large drop in A1C is needed. Thus a patient poorly-controlled with a low tolerance formedication would be the least suitable candidate for this class of medications. A recent review noted that there is noevidence that AGIs reduce morbidity or mortality.46<strong>American</strong> <strong>Diabetes</strong> <strong>Association</strong> <strong>Self</strong>-<strong>Assessment</strong> <strong>Program</strong>Educational (Learning) Critiques


Bibliography1. Nathan DM, et al: Management of hyperglycemia in type 2 diabetes: A consensus statement from the <strong>American</strong> <strong>Diabetes</strong> <strong>Association</strong>and the European <strong>Association</strong> for the Study of <strong>Diabetes</strong> Mellitus, <strong>Diabetes</strong> Care 2006;29:1963–1972.2. Van de Laar FA, et al: Alpha-glucosidase inhibitors for type 2 diabetes. The Cochrane Database of Systematic Reviews 2006 Issue 4.Item 49Answer CEach of the options listed (A, B, D, E), except option (C), is associated with significant insulin deficiency. Whetherthe fasting blood glucose level is high (250 mg/dL), the A1C is 10%, or the patient is symptomatic withpolyuria and polydypsia, the quickest way to stabilize the patient and reduce their blood glucose is by startinginsulin. When ketonuria is present it also indicates that the peripheral tissues are unable to utilize glucose as a fueland must resort to lipolysis. Often insulin can be started, quickly titrated to optimum levels and oral agents can beadded. As the blood glucose level corrects, eventually the insulin can often be withdrawn. The initiation of insulinoften is associated with a weight gain of 2–4 kg as the glycosuria is arrested and these calories are no longer lost.However, the specific body mass index should not be a criterion for deciding on the initiation of insulin therapy.Bibliography1. Nathan DM, et al: Management of hyperglycemia in type 2 diabetes: A consensus statement from the <strong>American</strong> <strong>Diabetes</strong> <strong>Association</strong>and the European <strong>Association</strong> for the Study of <strong>Diabetes</strong> Mellitus. <strong>Diabetes</strong> Care 2006;29:1963–1972.Item 50Answer AThe <strong>American</strong> <strong>Diabetes</strong> <strong>Association</strong> (ADA) and the European <strong>Association</strong> for the Study of <strong>Diabetes</strong> (EASD)recently issued a consensus statement regarding the management of hyperglycemia in patients with type 2 diabetes.In this statement the goals and principles of therapy were outlined and the various options for treatment discussed.There are many factors to consider when lifestyle modifications are inadequate to control the blood glucose level andthe decision to start a patient on medication has been made. Considerations include the goals of the patient, the anticipatedshort- and long-term benefits of therapy, and the short- and long-term risks, possible side-effects, expectedadherence to therapy, cost of the medication, insurance coverage and non-glycemic effects of the medication.The patient described in this item is a common one seen in clinical practice. She is overweight, hypertensive andrequires a significant reduction in her A1C level to be at her goal. Metformin is an excellent initial choice for herstarting medication as it is generally well tolerated, has a low incidence of hypoglycemia, and generally does not leadto additional weight gain. It is also available in a generic formulation, and is relatively effective in controlling bloodglucose.A sulfonylurea, while inexpensive, is more likely to cause hypoglycemia and additional weight gain. A glitazone hasthe advantage of possibly improving the patient’s lipid profile, but may cause water retention or weight gain andis much more expensive than either metformin or a sulfonylurea. Inhaled insulin is relatively new to the US market,is expensive and would not be considered as a first line medication in most patients. Sitagliptin, a DPP-IV inhibitorwhich acts upon the incretin system to increase insulin release, is also new to the market, expensive and too novel torecommend as starting therapy in most patients. For these reasons and others, the consensus panel recommendedmetformin as the initial choice of drug in most situations. “Metformin is recommended as the initial pharmacologictherapy, in the absence of specific contraindications, for its effect on glycemia, absence of weight gain or hypoglycemia,generally low level of side-effects, high level of acceptance, and relatively low cost.” Table 2 summarizesthe effects of multiple treatment options in diabetes.Educational (Learning) Critiques<strong>American</strong> <strong>Diabetes</strong> <strong>Association</strong> <strong>Self</strong>-<strong>Assessment</strong> <strong>Program</strong>47


Table 2. Summary of anti-diabetic interventions as monotherapy.Expected decreaseInterventions in A1C (%) Advantages DisadvantagesStep 1: Initial TherapyLifestyle todecrease weight andincrease activity 1–2 Low cost, many benefits Fails for most in first yearMetformin 1.5 Weight neutral, inexpensive GI side-effects, rare lactic acidosisStep 2: Additional TherapyInsulin 1.5–2.5 No dose limit,inexpensive, Injections, monitoring,improved lipid profilehypoglycemia, weight gain.Sulfonylureas 1.5 Inexpensive Weight gain, hypoglycemia*TZDs 0.5–1.4 Improved lipid profile Fluid retention, weight gain,expensiveOther Drugsα-Glucosidase inhibitors 0.5–0.8 Weight neutral Frequent GI side-effects,t.i.d. dosing, expensive.Exenatide 0.5–1.0 Weight loss Injections, frequent GI side-effects,expensive, little experienceGlinides 1–1.5 † Short duration t.i.d. dosing, expensivePramlintide 0.5–1.0 Weight loss Injections, t.i.d. dosing,frequent GI side-effects,expensive, little experience* Severe hypoglycemia is relatively infrequent with sulfonylurea therapy. The longer-acting agents (e.g., chlorpropamide, glyburide [glibenclamide], andsustained-release glipizide) are more likely to cause hypoglycemia than glipizide, glimepiride, and gliclazide. Repaglinide is more effective at loweringA1C than nateglinide.Bibliography1. Nathan DM, et al: Management of hyperglycemia in type 2 diabetes: A consensus statement from the <strong>American</strong> <strong>Diabetes</strong> <strong>Association</strong>and the European <strong>Association</strong> for the Study of <strong>Diabetes</strong> Mellitus. <strong>Diabetes</strong> Care 2006;29:1963–1972.Item 51Answer AThe consensus statement of the ADA and EASD recommends an algorithm for the introduction of insulinif that is deemed necessary (Figure 3). After allowing for patient characteristics, lifestyle, and meal schedules,it is recommended to start with 10 units or 0.2 units/kg of bedtime intermediate-acting insulin or a similar doseof long-acting insulin taken at bedtime or in the morning. A single dose of insulin is likely to have better complianceand by starting low and titrating up, the chances of hypoglycemia are minimized. Blood glucose levels should bemonitored at least daily and the dose adjusted frequently (typically by 2 units every 3 days) until a desired fastinglevel of blood glucose is obtained (70–130 mg/dL). If the fasting glucose is 180 mg/dL, a faster titration maybe initiated by increasing the dose by 4 units every 3 days.48<strong>American</strong> <strong>Diabetes</strong> <strong>Association</strong> <strong>Self</strong>-<strong>Assessment</strong> <strong>Program</strong>Educational (Learning) Critiques


okitsculnm omn ilgoerome igimeeouc ddos seaiebeskcrnagnsaeltidl-gomncudtsgngialcnratein egintiuntoubghpradiakeulrlauglittsiregocpgnisgacmrecnnraananid-enemrbatc-adiprcedunio narlaisuitttseintlalubggnb.pitnuuobgsatdadoe oeiaddmi stm msmolindecau2itnsnionadtulsitignhcrgahcgnlslhetanalsunnck ebonirgnadlgbrggbenralsl-eprsuetlnih twiueb-sindliunuadbgdblmsdnincoeitsnut odi ctnindrendStart with bedtime intermediate-acting insulinor bedtime or morning long-acting insulin.Can initiate with 10 units or 0.2 units per kgrcininacceCh fa e(f sco itrsek) y d ilyadnae, ty lypiby2 un evitry 3d ays lf a gniev eera rta g e(70– 130 mg/d L o r3 .89 –7. 2 2 ol/ L);esdots ,. g y 4 tinuev ery3d ays, ffas i se 180 g/ dL( 10 ol/ L)rfa(3.894urrts 1 0 do eIfh yp yc ucoc s,gg les ve 70 mg/dLl/L), de, %if 60A1 Cesstby7%aft eNO2– r 3 mYESntohs ?Iffa s tinet ra e( ng 70– 130 g/ dL)or 3.89–7. 22 m m /L),ch, -d er ,d d, -b des ,in jec on; n us lyua~4u nd ad ju by e ver y 3 day s ting in ra egntinCocehceve ryer ue3m non;A1ChstNOnraPree :arbtaniulsniof-a cfagnPreg e :aparernnkfaeotNPHt ortaglu nliufPreofg e :a d ra -pan atgniofoutRec hnrae,mif A1C2ckechhp --eprckendarntoA1 C7%af ert3YESl bg l sean dn ees to bst -p l eipra-a c niame-e ev ifyofr e,levn je tcjus ton; iFigure 3. Algorithm for starting insulin.Initiation and adjustment of insulin regimens. Insulin regimens should be designed taking lifestyle and meal schedule into account. The algorithm can onlyprovide basic guidelines for initiation and adjustment of insulin. Premixed insulins are not recommended during adjustment of doses; however, they can be usedconveniently, usually before breakfast and/or dinner if proportion of rapid- and intermediate-acting insulins is similar to the fixed proportions available.bg = blood glucose.Bibliography1. Nathan DM, et al: Management of hyperglycemia in type 2 diabetes: A consensus statement from the <strong>American</strong> <strong>Diabetes</strong> <strong>Association</strong>and the European <strong>Association</strong> for the Study of <strong>Diabetes</strong> Mellitus. <strong>Diabetes</strong> Care 2006;29:1963–1972.Educational (Learning) Critiques<strong>American</strong> <strong>Diabetes</strong> <strong>Association</strong> <strong>Self</strong>-<strong>Assessment</strong> <strong>Program</strong>49


Bibliography1. www.cdc.gov/nip/recs/adult-schedule.htm2. www.immunize.org/catg.d/4036need.pdf (screening tool)3. www.immunize.org/catg.d/p3075.pdf (standing orders for Pneumococcal vaccine)Item 56Answer CThe UK Prospective <strong>Diabetes</strong> Study (UKPDS) Group found that while type 2 diabetes is a progressive disease,on average, patients had already lost about half (50%) of their beta-cell function by the time they were diagnosed.This highlights the need to intervene with patients before frank diabetes is confirmed. Multiple interventionsincluding diet, exercise, weight loss and certain medications have been found to reduce the progression fromglucose intolerance to diabetes.Bibliography1. UK Prospective <strong>Diabetes</strong> Study Group. UK Prospective <strong>Diabetes</strong> Study 16. Overview of 6 years’ therapy of type 2 diabetes:a progressive disease. <strong>Diabetes</strong> 1995;44:1249–1258.Item 57Answer DHypoglycemia generally occurs gradually and is preceded by warning signs, such as anxiety, diaphoresis,tachycardia and shakiness. The level of serum glucose that causes symptoms of hypoglycemia may vary frompatient to patient and even within the same patient at times. Impaired level of consciousness may occur withhypoglycemia but death is uncommon, unless warning signs are ignored for a prolonged period or a seriousmedication error is made. Death from low blood glucose levels alone rarely occurs unless it involves a dangeroussituation, e.g., while driving, climbing stairs, etc.A recent study of adverse drug reactions that led to an emergency department visit showed that there are about700,000 such patients’ visits annually. One quarter of these visits were by patients over the age of 65 and almost3500 patients were admitted. The medications most commonly causing adverse events were insulins, oral hypoglycemics.opioid-containing products, anticoagulants, amoxicillin-containing products, and antihistamines/coldproducts. One-third of adverse events in elderly patients were caused by warfarin, insulin, and digoxin. Over 85%of elderly patients who presented with an unintentional overdose were taking anti-hyperglycemic agents, warfarin,anti-convulsants, digoxin, theophylline, or lithium.Bibliography1. <strong>American</strong> <strong>Diabetes</strong> <strong>Association</strong>. <strong>Diabetes</strong> Care 2007;30(Suppl 1):S85.2. Budnitz DS, et al: National surveillance of emergency department visits for outpatient adverse drug events. JAMA 2006;296:1858–66.Item 58Answer EInsulin actually promotes the storage of triglycerides into adipose tissue, not the degradation of triglycerides intoFFAs. Therefore, if there is insulin resistance, the uptake of triglycerides is reduced and more FFAs are present inthe blood stream. All the other options (A, B, C, D) are consistent with the expected effects of insulin resistance.The normal suppression of gluconeogenesis by insulin in the liver is impaired. The normal uptake of glucose byskeletal muscle is reduced leading to elevated glucose levels. The normal glucose disposal, which includes glucoseoxidation and glycogen synthesis, by skeletal muscle, is impaired, raising the glucose levels further. Mildly elevatedlevels of FFAs do not impair β-cell function initially but actually increase insulin secretion to compensate for theearly phase of insulin resistance. As the FFA levels rise however, β-cell dysfunction and damage occurs and thecompensatory ability to increase insulin production is overwhelmed and relative insulin deficiency occurs.Bibliography1. Unger RH. Lipotoxic diseases. Ann Rev Med 2002;53:319–336.52<strong>American</strong> <strong>Diabetes</strong> <strong>Association</strong> <strong>Self</strong>-<strong>Assessment</strong> <strong>Program</strong>Educational (Learning) Critiques


Item 59Answer EEnvironmental factors, such as diet and exercise, clearly impact the development of pancreatic β-cell dysfunction.Genetic factors, such as family history of diabetes or glucose intolerance increase the chances significantly of developingthe same conditions. Type 2 diabetes is 2–6 times more prevalent in African-<strong>American</strong>s, Native <strong>American</strong>s,Pima Indians, and Hispanic <strong>American</strong>s in the US than in white <strong>American</strong>s. In addition, 39% of patients with type 2diabetes have at least one parent with the disease. As discussed in the critique for Item 60, FFAs contribute topancreatic β-cell dysfunction by direct toxicity to the islet cells. Studies also suggest that glucose itself furtherincreases β-cell loss by several mechanisms including protein glycation and glucokinase down-regulation, leadingto reduced production of insulin and even β-cell death. There does not appear to be any direct insulin toxicitywhich contributes to pancreatic β-cell dysfunction.Bibliography1. Unger RH. Lipotoxic diseases. Ann Rev Med 2002;53:319–3362. Klein BE, et al: Parental history of diabetes in a population-based study. <strong>Diabetes</strong> Care 1996;19:827.3. Harris MI, et al: Prevalence of diabetes, impaired fasting glucose, and impaired glucose tolerance in U.S adults: the Third NationalHealth and Nutrition Examination Survey, 1988–1994. <strong>Diabetes</strong> Care 1998;21:518.Item 60Answer COption (A) This is diabetic dermopathy which often occurs as pigmented, pretibial, symmetric pink andbrown-bronze atrophic shiny skin patches. In this picture we see the leg of a 59-year-old man withdiabetes. Although this condition is not specific to diabetes, it is probably the most common findingin the skin of diabetics. These pigmented pretibial patches may be a marker for underlying cutaneousmicroangiopathy.Option (B) Acanthosis nigricans is a pigmenting disorder which causes velvety, light-brown-to-black markingsusually on the neck, under the arms, or in the groin. Acanthosis nigricans is most often associated withobesity and diabetes. It is thought to be caused by elevated insulin levels during the early phase ofinsulin resistance which activates insulin receptors in the skin, causing it to be more heavily pigmented.Option (C) This is a case of scabies which is not specifically related to diabetes.Option (D) This is a diabetic foot ulcer. These develop from a combination of neuropathy causing an underestimationof pressure on an area of the foot, and microangiopathy, poor blood flow and wound healing.Bibliography1. All of the images were borrowed with permission from:Cohen BA, et al: DermAtlas. Johns Hopkins University, 2006. www.DermAtlas.orgItem 61Answer EPatients who develop type 2 diabetes mellitus have a progressive decline in β-cell function. The UKPDS showedthat at the time of diagnosis of type 2 diabetes mellitus one has lost 50% of β-cell function and continues to loseβ-cell function over time (Figure 5). The graph thus suggests that loss of β-cells may accrue for up to 10 yearsprior to the diagnosis of type 2 diabetes mellitus. This estimate is confirmed by retinopathy rates that exist at timeof diagnosis of type 2 diabetes mellitus compared to time known to develop retinopathy in type 1 diabetes wheretime of onset is more precisely defined (NHANES III). The question had been raised that if one screened for type2 diabetes mellitus in patients at risk, before they have presumably lost 50% of their β-cells, and found patientswith IGT, might one delay or prevent the onset of overt diabetes? Many studies, pictorialized in Figure 6, now haveshown reduced risk of developing type 2 diabetes mellitus with lifestyle changes, and with metformin in the DPP4study, with acarbose in the Stop-NIDDM trial and with rosiglitazone in the DREAM trial. However in the DREAMtrial as well, ramipril was not shown to delay or prevent type 2 diabetes mellitus. The ADA recently recommendeda combination of lifestyle changes and metformin for patients with IFG and IGT.Educational (Learning) Critiques<strong>American</strong> <strong>Diabetes</strong> <strong>Association</strong> <strong>Self</strong>-<strong>Assessment</strong> <strong>Program</strong>53


β-cell function (% of normal by HOMA)100?80604020Time of diagnosisPancreatic function = 50% of normal0-10 -9 -8 -7 -6 -5 -4 -3 -2 -1 0 1 2 3 4 5 6YearsFigure 5. Decline of β-cell function in the UKPDS illustrates progressive nature of diabetes.<strong>Diabetes</strong> Mellitus Reduction (%)70605040302010058% 58%Finnish42%Da Qing –Diet + ExerciseDPP-Lifestyle31%DPP-Metformin25%STOP-NIDDM<strong>Diabetes</strong> Prevention Clinical Trials55% 55%TRIPOD41%XENDOS62%DREAMPIOPODFigure 6. Is it possible to delay the onset of type 2 diabetes mellitus?Bibliography1. Wajchenberg, BL. β-Cell Failure in <strong>Diabetes</strong> and Preservation by Clinical Treatment. Endocr Rev 2007;28(2):187–218.2. Gillies. Pharmacological and lifestyle interventions to prevent or delay type 2 diabetes in people with IGT. BMJ 2007;334:299–302.3. Jeon C. Physical Activity of Moderate Intensity and Risk of Type 2 <strong>Diabetes</strong>. <strong>Diabetes</strong> Care 2007;30:744–752.4. Henness S. Pharmacologic interventions in the prevention of Type 2 <strong>Diabetes</strong>. Clin Op in Endocrinology, <strong>Diabetes</strong> And Obesity2007;14:166–169.5. Holman RR. Diab Res Clin Pract 1998;40(suppl):S21–S25.6. UKPDS. <strong>Diabetes</strong> 1995;44:1249–1258.7. Nathan DM, et al: <strong>Diabetes</strong> Prevention <strong>Program</strong> (DPP). N Engl J Med 2002;346:393–403.8. Chiasson JL, et al: Prevent Non-Insulin-Dependent <strong>Diabetes</strong> Mellitus (NIDDM). Lancet 2002;359:2072–77.9. Buchanan T, et al: Troglitazone in the Prevention of <strong>Diabetes</strong> (TRIPOD). <strong>Diabetes</strong> 2002;51(9):27960–2803.10. Torgerson JS, et al: Xenical in the Prevention of <strong>Diabetes</strong> in Obese Subjects (XENDOS). <strong>Diabetes</strong> Care 2004;27(1):155–61.11. Gerstein H, et al: <strong>Diabetes</strong> Reduction <strong>Assessment</strong> with Ramipril & Rosiglitazone Medication (DREAM). Lancet 2006;368:1096–1105and NEJM 2006;355:1551–1562.12. Nathan, et al: Impaired Fasting Glucose and Impaired Glucose Tolerance (PIOPOD), implications for care. <strong>Diabetes</strong> Care2007;30:753–759.13. Harris MI, et al: Is the risk of diabetic retinopathy greater in non-Hispanic blacks and Mexican <strong>American</strong>s than in non-Hispanic whiteswith type 2 diabetes? A U.S. population study. <strong>Diabetes</strong> Care 1998;21(8):1230–5.54<strong>American</strong> <strong>Diabetes</strong> <strong>Association</strong> <strong>Self</strong>-<strong>Assessment</strong> <strong>Program</strong>Educational (Learning) Critiques


Item 62Answer DIt is widely believed that a meal plan that focuses on the foods the patient is used to eating is most likely to engenderthe best compliance. This also fits with the concept that there is no specific prescription for proportion of carbohydrate,fat, and protein combinations that are specifically proven to result in better care of the patient with diabetes.This also implies that if one tries to change the foods the patient is used to eating, then, by definition they will, mostlikely, not be able to follow the diet even in the short-term, or especially the long-term. So nutrition counseling thatis individualized holds the best chance for success in fulfilling the Nutrition recommendations of the ADA.Extracts from ADA recommendation:“The Dietary Reference Intakes (DRI) report of the Institute of Medicine recommends that, to meet thebody’s daily nutritional needs while minimizing risk for chronic diseases, healthy adults should consume45–65% of total energy from carbohydrate, 20–35% from fat, and 10–35% from protein. It must be clearlyrecognized that regardless of the macronutrient mix, total caloric intake must be appropriate to weightmanagement goals.”… Sugar alcohols and non-nutritive sweeteners are safe when consumed within the dailyintake levels established by the FDA… The FDA has approved five non-nutritive sweeteners for use in theU.S. These are acesul fame potassium, aspartame, neotame, saccharin, and sucralose. Before being allowedon the market, all underwent rigorous scrutiny and were shown to be safe when consumed by the public,including people with diabetes and women during pregnancy. Substantial evidence from clinical studiesdemonstrates that dietary sucrose does not increase glycemia more than isocaloric amounts of starch.[Patients with] “type 2 diabetes should be encouraged to achieve the US Department of Agriculture (USDA)recommendation for dietary fiber (14 g fiber/1000 kcal)” …A dietary pattern that includes carbohydratefrom fruits, vegetables, whole grains, legumes, and low-fat milk is encouraged for good health.Bibliography1. Institute of Medicine: Dietary Reference. Intakes: energy, carbohydrate, fiber, fat, fatty acids, cholesterol, protein, and amino acids.National Academies Press, Washington, DC. 2002.2. Nutrition Recommendations and Interventions for <strong>Diabetes</strong>. <strong>Diabetes</strong> Care 2007;30(Suppl 1):S48–S65.3. Franz MJ, et al: Evidence-based nutrition principles and recommendations for the treatment and prevention of diabetes and relatedcomplications. <strong>Diabetes</strong> Care 2002;25:148–198.Item 63Answer AA characteristic syndrome has been described in African-<strong>American</strong>s, often seen in patients with new-onset diabeteswho present with DKA. After initial usual aggressive care of their DKA and its precipitant, they are able to beweaned from insulin and act as typical type 2 patients. It is felt to be a genetically specific atypical diabetes, mostlikely related to the ability of the β-cell to recover insulin secretion function, possibly initially suppressed by anundue glycotoxicity or lipotoxicity. African-<strong>American</strong>s do not have any particular extra responsiveness to TZDs.They have an increased incidence of family history of diabetes, as do other patients with type 2 diabetes. There isno association with sickle cell trait and the expected low proportions of type 2 patients who have anti-islet cellantibodies. Option (A) is the correct answer, the ability to stop insulin therapy soon after presentation with DKA.Bibliography1. Banerji MA, et al: GAD antibody negative NIDDM in adult black subjects with diabetic ketoacidosis and increased frequency of humanleukocyte antigen DR3 and DR4. Flatbush diabetes. <strong>Diabetes</strong> 1994;43:741–745.2. Pinero-Pilona A, et al: Idiopathic type 1 diabetes. J <strong>Diabetes</strong> Complications 2001;15:328–335.3. Maldonado M, et al: Ketosis-prone diabetes: dissection of a heterogeneous syndrome using an immunogenetic and ß-cell functionalclassification, prospective analysis, and clinical outcomes. J Clin Endocrinol Metab 2003;88:5090–5098.4. Kitabchi AE. Ketosis-prone diabetes: a new subgroup of patients with atypical type 1 and type 2 diabetes? J Clin Endocrinol Metab2003;88:5087–5089.5. Wajchenberg BL. β-Cell Failure in <strong>Diabetes</strong> and Preservation by Clinical Treatment. Endocr Rev 2007;28(2):187–218.Educational (Learning) Critiques<strong>American</strong> <strong>Diabetes</strong> <strong>Association</strong> <strong>Self</strong>-<strong>Assessment</strong> <strong>Program</strong>55


Item 64Answer CThe ADA believes a A1C less than 7% is a reasonable goal for glycemic control. A preprandial capillary plasmaglucose level should be 90–130 mg/dL and peak post-prandial capillary plasma glucose 180 mg/dL. The ADAalso recommends that A1C goals should be as low as possible, even 6%, but, most importantly, as low as possiblewithout undue hypoglycemia. So in this patient option (C) would be the correct answer, a fasting glucose level140 mg/dL would not be considered low enough.The discussion in consensus conferences had to do with the accepted data that damage to tissues starts withpost-prandial glucose levels 140 mg/dL (DECODE, Honolulu Heart Study), and with DCCT, UKPDS, andEDIC trials we know that decrease in A1C reduces risks of complications, but that risk of hypoglycemia, especiallywith recommended therapies of secretogogues and insulin, engender significant risks of hypoglycemia. So at asocietal level, one would not want to cause undue hypoglycemia in a whole population because of goals that mightbe too “tight.”Table 3. Intensive therapy for diabetes: Reduction in incidence of complications.Type 1 Type 2 Type 2DCCT Kumamoto UKPDSA1C 9 7% 9 7% 8 7%Retinopathy 63% 69% 17–21%Nephropathy 54% 70% 24–33%Neuropathy 60% – –Cardiovascular disease 41% – 16%DCCT Research Group. N Engl J Med 1993;329:977–986.Ohkubo Y, et al: <strong>Diabetes</strong> Res Clin Pract 1995;28:103–117.UKPDS 33: Lancet 1998;352:837–853.Adapted from D. Kendall, International <strong>Diabetes</strong> Center56<strong>American</strong> <strong>Diabetes</strong> <strong>Association</strong> <strong>Self</strong>-<strong>Assessment</strong> <strong>Program</strong>Educational (Learning) Critiques


DCCT A1C 9.2% vs 7.1% forMean 6.5 years; EDICT avg A1C 8.00.120.100.080.060.040.02Any Cardiovascular EventConventionalIntensive0.000 1 2 3 4 5 6 7 8 9 10 11 12 13 14 15 16 17 18 19 20 21Years Since EntryNumber at RiskIntensive: 705 683 629 113Conventional: 714 688 618 9242% Risk ReductionP + 0.0170.12Non-Fatal MI, Non-Fatal Stroke or Cardiovascular DeathCumulative Incidence0.100.08Figure 7. DDCT/EDIC Trial: Reduction in CV outcomes by glycemic control with insulin therapy.Bibliography0.06Conventional0.040.0258% Risk ReductionP = 0.18Intensive0.000 1 2 3 4 5 6 7 8 9 10 11 12 13 14 15 16 17 18 19 20 21Years Since EntryNumber at RiskIntensive: 705 686 640 113Conventional: 721 694 637 961. Hanefeld M, et al: Diabetologia 1996;39:1577–1583.2. Decode Study Group. Lancet 1999;354:617–21.3. Donahue RP, et al: <strong>Diabetes</strong> 1987;36:689–692.4. DCCT Research Group. The effect of intensive diabetes treatment on the development and progression of long-term complicationsin insulin-dependent diabetes mellitus: <strong>Diabetes</strong> Control and Complications Trial (DCCT). N Eng J Med 1993;329:977–86.5. Harris MI, et al: Is the risk of diabetic retinopathy greater in non-Hispanic blacks and Mexican <strong>American</strong>s than in non-Hispanic whiteswith type 2 diabetes? A U.S. population study. <strong>Diabetes</strong> Care 1998;21(8):1230–5.6. <strong>American</strong> <strong>Diabetes</strong> <strong>Association</strong>. Clinical Practice Recommendation. 1999.7. DCCT Group. <strong>Diabetes</strong> Care 1995;44:968–983.8. DCCT/EDIC Research Group. The effect of intensive diabetes treatment of cardiovascular disease in type 1 diabetes:The <strong>Diabetes</strong> Control and Complications Trial/Epidemiology of <strong>Diabetes</strong> Interventions and Complications (EDIC) Study.N Eng J Med 2005;353:2643–2653.9. United Kingdom Prospective <strong>Diabetes</strong> Study (UKPDS) Group. Intensive blood glucose control with sulphonylureasor insulin compared with conventional treatment and risk of complications in patients with type 2 diabetes (UKPDS 33).Lancet 1998;352:837–53.Educational (Learning) Critiques<strong>American</strong> <strong>Diabetes</strong> <strong>Association</strong> <strong>Self</strong>-<strong>Assessment</strong> <strong>Program</strong>57


Item 65Answer EThe current ADA guidelines suggest metformin should be started at the onset of therapy for all the reasonsmentioned in this item. It improves insulin resistance in liver, decreasing gluconeogenesis and glycogenolysis. It has a high initialresponse rate, drops A1C 2%. There is no initial weight gain and can have a modest initial weight loss. It is not associated with hypoglycemia when used alone or with TZD, exenatide, or DPP-4 Inhibitors. It has potential to delay or prevent type 2 diabetes mellitus and progression (DPP-2 trial), but its secondaryfailure is only modestly better than sulfonylureas (ADOPT trial). It decreases MIs 39% in the retrospective analysis in the UKPDS obese subgroup, it decreases advancedglycosylated end-products (AGEs), and improves endothelial dysfunction. Disadvantages – One must watch for: GI side-effects on initiation, though titration up to maximal effective dose of 2 g/day over a monthdecreases the risk. Delay initiation until Cr levels are proven stable using radiologic iodinated contrast media studies. This willreduce risk of lactic acidosis. Moreover, don’t use if have significantly impaired hepatic function or cardiovascularcompromise (CHF) or Cr 1.4 female, 1.5 male. Blood levels of metformin increase in patients with Cr clearance 70 (age 70). Patients with Cr Clearance40 is where most cases of lactic acidosis cases have occurred. Thus, clinical judgement must be used inpatients age 70–80 (the age the product insert recommends stopping or not using metformin).iD-Glucos e+Prote n(”R” ) Schiff Ba seHO CHO HO ++RNH2 NHRHOHOOHOHOHOHOHOHdori maAPro uct dONHRHOOHOHOHAP- e-d enenoiOHOOH[Pro te i n]NHROdA.G .E.Cro ssli nke Pr ote nsiHNOOX [Pro te i n]OHFigure 8. Advanced glycation end products.Aminoguanidine (AMG)NHNH 2 C NH NH 2AminoguanidineNHNHNH 2 C NH C NMetformin(Dimethylbiguanide)CH 3CH 3Figure 9. Guanidino compounds that bind dicarbonyls.58<strong>American</strong> <strong>Diabetes</strong> <strong>Association</strong> <strong>Self</strong>-<strong>Assessment</strong> <strong>Program</strong>Educational (Learning) Critiques


FructoselysineSerineTriose PhosphatesDihydroxyacetone PO 4Glyceraldehyde-3-PO 4GAPDHGlucoseL-LactatePyruvateKetone BodiesAcetoacetateβ-HydroxybutyrateEne-diolMethylglyoxalMono-oxygenaseAcetolMono-oxygenaseP = 450AcetoneAminoacetoneL-ThreonineDehydrogenaseSSAOFigure 10. Sources of methylglyoxal.Media Methylglyoxal(pmols/gm RBCs)40003500300025002000150010005000P = 0.03+ Metformin - MetforminTreatment GroupFigure 11. Media levels of methylglyoxal with or without metformin.Bibliography1. Nathan DM, et al: Management of hyperglycemia in type 2 diabetes: A consensus algorithm for the initiation and adjustment of therapy.A consensus statement from the <strong>American</strong> <strong>Diabetes</strong> <strong>Association</strong> and the European <strong>Association</strong> for the Study of <strong>Diabetes</strong>. <strong>Diabetes</strong> Care2006;29:1963–1972.2. Bailey CJ, et al: Metformin. N Engl J Med 1998;334:574–583.3. DeFronzo R, et al: Metformin Study Group. Efficacy of metformin in patients with non-insulin-dependent diabetes mellitus.N Engl J Med 1995;333:541.4. <strong>Diabetes</strong> Prevention <strong>Program</strong> Research Group: Reduction in the incidence of type 2 diabetes with lifestyle intervention or metformin.N Engl J Med 2002;346:393–403.5. Kahn SE, et al: ADOPT trial. N Engl J Med 2006;355:2427–43.6. UKPDS Group. Effect of intensive blood-glucose control with metformin on complications in overweight patients with type 2 diabetes(UKPDS 34). Lancet 1998;352:854–865.7. Rahbar S, et al: Evidence that pioglitazone, metformin and pentoxifylline are inhibitors of glycation. Clin Chem Acta2000;301(1–2):65–77.8. Ruggiero-Lopez D, et al: Reaction of metformin with dicarbonyl compounds. Possible implication in the inhibition of advancedglycation end product formation. Biochem Pharmacol 1999;58(11):1765–73.9. Vitale C, et al: Metformin improves endothelial function in patients with metabolic syndrome. J Intern Med 2005;258(3):250–6.Educational (Learning) Critiques<strong>American</strong> <strong>Diabetes</strong> <strong>Association</strong> <strong>Self</strong>-<strong>Assessment</strong> <strong>Program</strong>59


Item 66Answer EMetformin reduces insulin levels and therefore it would reduce C-peptide levels. In fact, metformin does reducehepatic glucose output, hepatic gluconeogenesis, insulin levels, and fasting blood glucose levels.Bibliography1. Macfarlane DP, et al: Oral antidiabetic agents as cardiovascular drugs. <strong>Diabetes</strong> Obes Metab 2007;9(1):23–30.2. Chan. Role of metformin in the initiation of pharmacotherapy for type 2 diabetes: an Asian-Pacific perspective. <strong>Diabetes</strong> Res Clin Prac2007;75(3)255–66.3. Bailey CJ, et al: Metformin. N Eng J Med 1996;334:574.4. Schimmack G, et al: AMP-activated protein kinase: Role in metabolism and therapeutic implications. <strong>Diabetes</strong> Obes Metab2006;8(6):591–602.5. Guigas B, et al: 5-Aminoimidazole-4-carboxamide-1-beta-D-ribofuranoside and metformin inhibit hepatic glucose phosphorylationby an AMP-activated protein kinase-independent effect on glucokinase translocation. <strong>Diabetes</strong> 2006;55(4):865–74.6. Natali A, et al: Effects of metformin and thiazolidinediones on suppression of hepatic glucose production and stimulation of glucoseuptake in type 2 diabetes: a systematic review. Diabetologia 2006;49(3):434–41.Item 67Answer BA general principle of combination therapy for treatment of hyperglycemia in type 2 diabetes without usinginsulin is that each agent will reduce A1C an additional 1%. There is a logic, though no published studies, to usea combination of TZD, metformin and an incretin mimetic to control blood glucose levels avoiding an undue riskof hypoglycemia. One achieves benefits of glycemic control, potential CV benefit, potential β-cell preservation,as well as decreased endothelial dysfunction and inflammation. Moreover, if exenatide is used, it is likely to achievesignificant weight loss. There is no added benefit in using two TZDs (rosiglitazone and pioglitazone).Bibliography1. Davidson MB. Triple therapy: definitions, application, and treating to target. <strong>Diabetes</strong> Care 2004;27(7):1834–5.2. Hamid Z, et al: Triple therapy in type 2 diabetes: insulin glargine or rosiglitazone added to combination therapy of sulfonylurea plusmetformin in insulin-naive patients: response to Rosenstock, et al. <strong>Diabetes</strong> Care 2006;29(10):2331.3. Dorkhan M, et al: Glycaemic and nonglycaemic effects of pioglitazone in triple oral therapy of patients with type 2 diabetes.J Intern Med 2006;260(2):125–33.4. Tran MT, et al: Comparison of the glycemic effects of rosiglitazone and pioglitazone in triple oral therapy in type 2 diabetes.<strong>Diabetes</strong> Care 2006;29(6):1395–6.5. Rosenstock J, et al: Triple therapy in type 2 diabetes: insulin glargine or rosiglitazone added to combination therapy of sulfonylureaplus metformin in insulin-naive patients. <strong>Diabetes</strong> Care 2006;29(3):554–9.6. Roberts VL, et al: Triple therapy with glimepiride in patients with type 2 diabetes mellitus inadequately controlled by metforminand a thiazolidinedione: results of a 30-week, randomized, double-blind, placebo-controlled, parallel-group study. Clin Ther2005;27(10):1535–47.Item 68Answer CIncreased quantity of visceral fat is felt to potentiate, by several different mechanisms, muscle and liver insulinresistance, reduce insulin secretion and is also is directly related to increasing vascular endothelial dysfunction.In the past, it was felt that insulin resistance was an issue purely of muscle with reduced glucose uptake, and liver withincreased glucose production; but several years ago, it was learned it is about fat, not fat in the buttocks and thethighs, but visceral obesity, the apple physiognomy, the beer belly, the increased visceral fat that accounts for increasedinsulin resistance, and is also associated with diabetes and its complications. When the individual fat cell increases insize and the total visceral fat quantity increases in volume, they become endocrine organs. They secrete cytokines andtoo much of some hormones like angiotensinogen, and too little of others like adiponectin. This affects musclereducingglucose uptake, liver-increasing glucose production, and actually affects the pancreas in reducing furtherβ-cell secretion. Moreover, the body attempts to reduce that volume and quantity of visceral fat by shifting free fattyacids to the periphery; and the free fatty acids also go to the muscle, reduced glucose uptake, go to the liver increasing60<strong>American</strong> <strong>Diabetes</strong> <strong>Association</strong> <strong>Self</strong>-<strong>Assessment</strong> <strong>Program</strong>Educational (Learning) Critiques


glucose production, also, in fact, reduce β-cell function even more and starts the diabetic state. Moreover, thecombination of FFAs, cytokines, and abnormal hormone secretions are directly related to endothelial dysfunctionand inflammatory changes in the arterial epithelium; and this is exacerbated by CRP and PAI-1 production by theinsulin-resistant liver, which all go to the lining of the arterioles and start the atherosclerotic process. Thus, there isa clear relationship between obesity, insulin resistance, diabetes and atherosclerotic disease.Bibliography1. Rader DJ. Effect of insulin resistance, dyslipidemia, and intra-abdominal adiposity on the development of cardiovascular diseaseand diabetes mellitus. Am J Med 2007;120(3 Suppl 1).2. Bergman RN, et al: Abdominal obesity: role in the pathophysiology of metabolic disease and cardiovascular risk.Am J Med 2007;120(2 Suppl 1):S3–8.Item 69Answer BPost-prandial hyperglycemia is the earliest detectable glycemic abnormality in the majority of patients withdiabetes; 80% of them present in this way. It contributes to the A1C level of 6.5% to a greater extent than doesfasting blood glucose. It has been associated with an increased risk of microvascular complications in theDECODE study and Honolulu heart study. It has been associated with an increased risk of macrovascular complicationsin the NHANES study as well as an interpretation of the DCCT results. Even with A1C at goal, 75% ofpost-prandial glucose levels will be elevated and, indeed, with A1C of 6.5% roughly 75% of this value is a resultof post-prandial hyperglycemia. There is one study that suggests that by controlling post-prandial glucose levelsadverse outcomes may be reduced i.e., complications of pregnancy in women with diabetes (decrease large fordatesbabies by 2/3, decrease C-sections by 2/3, and almost eliminate neo-natal hypoglycemia). However, otherthan that study, there is no data that proves that reducing post-prandial hypoglycemia will, in fact, reduce othercomplications or cardiovascular outcomes. So, the correct answer is option (B), that no convincing data exists thattreating post-prandial glucose elevations will reduce cardiovascular outcomes or other complications of diabetes.Bibliography1. Yamagishi I, et al: Role of post-prandial hyperglycaemia in cardiovascular disease in diabetes. Int J Clin Prac 2007;61(1)83–87.2. Rendell MS, et al: Targeting post-prandial hyperglycemia. Metabolism 2006;55(9):1263–81.3. Leiter LA, et al: International Prandial Glucose Regulation (PGR) Study Group. Post-prandial glucose regulation: New data and newimplications. Clin Ther 2005;27(Suppl 2):S42–56.4. Robertson C. Physiologic insulin replacement in type 2 diabetes: optimizing post-prandial glucose control. <strong>Diabetes</strong> Edu2006;32(3):423–32.5. Brindisi MC, et al: Post-prandial hyperglycaemia: to treat or not to treat? <strong>Diabetes</strong> Metab 2006;32(2):105.6. Gerich JE. Post-prandial hyperglycemia and cardiovascular disease. Endocr Pract 2006;12(Suppl 1):47–51.7. Mecacci F, et al: Maternal metabolic control and perinatal outcome in women with gestational diabetes treated with regular or lisproinsulin: comparison with non-diabetic pregnant women. Eur J Obstet Gynecol Reprod Bio 2003;111:19–24.8. de Veciana M, et al: Post-prandial versus pre-prandial blood glucose monitoring in women with gestational diabetes mellitus requiringinsulin therapy. N Engl J Med 1995;333:1237–1241.Item 70Answer CFast analog insulins were developed to mimic normal physiology of prandial insulin, release and duration. Studieshave shown that they can reduce hypoglycemia overnight or during the day. They can decrease post-prandialhypoglycemia by its rapid onset and loss of effect over several hours. As part of a basal/bolus protocol of insulintherapy, it allows the patient to eat when they wish, skip or delay meals, eat varying quantity of meals, do shift work,travel across time zones, wake up different times on weekends and weekdays and in that sense instead of the diseasetherapy controlling ones lifestyle, i.e., must eat at the same time everyday, the patient can have freedom of lifestyle,have control over their disease. Though we would have liked to have seen studies confirming reduction consistentlyin A1C, this has not occurred as yet in the published studies.Educational (Learning) Critiques<strong>American</strong> <strong>Diabetes</strong> <strong>Association</strong> <strong>Self</strong>-<strong>Assessment</strong> <strong>Program</strong>61


Bibliography1. Robertson C. Physiologic insulin replacement in type 2 diabetes: optimizing post-prandial glucose control. <strong>Diabetes</strong> Edu2006;32(3):423–32.2. Mudaliar SR, et al: Insulin aspart (B28 asp-insulin): a fast-acting analog of human insulin: absorption kinetics and action profilecompared with regular human insulin in healthy nondiabetic subjects. <strong>Diabetes</strong> Care1999;22(9):1501–6.3. Heller SR, et al: Effect of the fast-acting insulin analog lispro on the risk of nocturnal hypoglycemia during intensified insulin therapy.U.K. Lispro Study Group. <strong>Diabetes</strong> Care 1999;22(10):1607–11.4. Bruttomesso D, et al: Restoration of early rise in plasma insulin levels improves the glucose tolerance of type 2 diabetic patients.<strong>Diabetes</strong> 1999;48(1):99–105.5. Dailey G, et al: Insulin glulisine provides improved glycemic control in patients with type 2 diabetes. <strong>Diabetes</strong> Care 2004 ;27(10):2363–8.Item 71Answer AMultiple metabolic mechanisms have now been implicated in the etiology of complications due to hyperglycemia.Strong data exists in the literature for the role of increased flux in the aldose reductase pathway, productionof advanced glycosylation end products, increased protein kinase C pathway flux, increase flux through thehexosamine pathway and hyperglycemia increasing oxidative stress in mitochondria. As a result they have multipleaffects in cells and in vascular endothelium and have been strongly associated with the production of eye, kidney,and nerve disease.There is also a pentose-phosphate shunt pathway that takes fructose 6-phosphate and creates, via transketolase,pentose-phosphate. One can give a medicine, benfotiamine, a fat-soluble thiamine relative, that will increasepentose-phosphate shunt flux and in doing so will reduce flux in several of the above pathways. Thus, the correctanswer is increasing pentose-phosphate shunt flux – it can have the potential to reduce complications of diabetes,not increase them.Pentose-5-phosphates+Erythrose-4-phosphateFigure 12.TKThiamine GlucoseNADPH NADP+ NAD+ NADH Sorbitol Fructose GAPDH O 2 Glucose-6-P Fructose-6-PPolyol pathwayGFAT Glucosamine-6-P UDP-GlcNAcGln GluHexosamine pathwayNADH NAD+ DHAP α-Glycerol-P DAG PKC Glyceraldehyde-3-PProtein kinase C pathwayNAD+ Methylglyoxal AGEsNADHAGE pathway1, 3-DiphosphoglycerateBibliography1. Brownlee M. The pathobiology of diabetic complications: A unifying mechanism. <strong>Diabetes</strong> 2005;54.2. Brownlee M. Biochemistry and molecular cell biology of diabetic complications. Nature 2001;414:813–820.62<strong>American</strong> <strong>Diabetes</strong> <strong>Association</strong> <strong>Self</strong>-<strong>Assessment</strong> <strong>Program</strong>Educational (Learning) Critiques


Item 72Answer DImproved outcomes in patients with diabetes are seen when the blood pressure is under 130/80 mm Hg.This was seen in the UK PDS and HOT trials. The recommendation of an ACE-inhibitor (ACE-I) or an ARB isfelt to be important for their additional benefit in reducing microalbuminuria if it exists. Patients with diabetes mayrequire combination therapy with an ACE-I or an ARB. Although beta-blockers are often needed especially in thepost MI state, it may increase the risk of hypoglycemic unawareness and, in the patient with peripheral vasculardisease, by leaving leaving unopposed alpha sympathetic activity, can result in new symptomatic claudication; thusthe physician must alert the patient in this regard. HCTZ can, in fact, increase blood glucose level but this is felt tobe a relative contraindication and not an absolute contraindication as option (D) states. Thus, the correct answer isoption (D). Thus clinically, in a patient with newly-diagnosed diabetes, one might avoid HCTZ as it may make thedifference between lifestyle therapy and need for monotherapy for glycemic control or between mono and combinationtherapy for glycemic control. However, in patients already on combination or insulin therapy, its effect onglycemic control is felt to be minimal and can easily be adjusted.Bibliography1. Hansson L, et al: HOT trial. Lancet 1998;351:1755–1762.2. UKPDS Group. BMJ 1998;317:703–713.3. Goodman, et al: Textbook of pharmacology [re: sulfonylurea and beta-blocker discussion].4. Russo D, et al: Am J Kidney Dis 1999;33:851–856.5. Parving HH, et al: N Engl J Med 2001;345:870–888.6. Viberti GC, et al: Circulation 2002;106:672–678.7. Lewis EJ, et al: N Engl J Med 2001;345:851–860.8. Brenner BM, et al: N Engl J Med 2001;345:861–869.Items 73–75Answers 73 (B); 74 (A); 75 (C)Morbidly obese patients would certainly benefit from weight loss. Weight loss has been shown to decrease bloodglucose levels, decrease insulin resistance, and decrease correlates of insulin resistance including endothelialdysfunction, inflammatory markers, dysfibrinolysis, insulin levels, blood pressure, and dyslipidemia. Accordingto 2-year published data (Figure 13), exenatide is the only drug for diabetes that seems to have a persistent effecton reduction in weight. In 30 weeks of treatment, 25% of patients lost 5–10 lbs, 25% lost 10–20 lbs and 25% lost20 lbs. Weight loss seems to continue for at least 2 years.In general, each additional agent in combination therapy of patients with type 2 diabetes drops the A1C 1% point.Thus, even if 2 additional agents were added in the patient with a A1C of 9.6%, already on metformin, it would notbring the A1C to even ADA goals of control. Thus, insulin, as the most “effective agent” to reduce blood glucose,is the best option in this example.A long-distance truck driver with diabetes on insulin therapy can no longer hold a commercial driver’s license.In addition, this truck driver is fearful of injections. Thus, a DPP-4 inhibitor, such as sitagliptin, would be the bestchoice with this patient.Educational (Learning) Critiques<strong>American</strong> <strong>Diabetes</strong> <strong>Association</strong> <strong>Self</strong>-<strong>Assessment</strong> <strong>Program</strong>63


00Change in Body Weight (kg)-1-2-3-4Placebo -control ledtrial sOpen-label xt ssionen(10 µg bid EXE)82-w eek complet er (n=314)Int ent-to-treat (n=5 51)-3.5 ± 0.2 kgChange in Body Weight (kg)-2-4-6-8


ADA-SAP 1 PDF Version – Multiple Choice Questions – Section A1 26 51 762 27 52 773 28 53 784 29 54 795 30 55 806 31 56 817 32 57 828 33 58 839 34 59 8410 35 60 8511 36 61 8612 37 62 8713 38 63 8814 39 64 8915 40 65 9016 41 66 9117 42 67 9218 43 68 9319 44 69 9420 45 70 9521 46 71 9622 47 72 9723 48 73 9824 49 74 9925 50 75 100<strong>American</strong> <strong>Diabetes</strong> <strong>Association</strong> <strong>Self</strong>-<strong>Assessment</strong> <strong>Program</strong>65© 2007 Professional Evaluation, Inc. All rights reserved. Please see page 66 for <strong>Program</strong> Evaluation Form


ADA-SAP 1 <strong>Program</strong> FeedbackThank you for participating in the ADA-SAP Module 1: Basic Principles of Diagnosis and Management of Type 2<strong>Diabetes</strong>. If you would like to provide any feedback on this or other <strong>Self</strong>-<strong>Assessment</strong> <strong>Program</strong>s from PEI, pleaseemail your comments to info@proevalinc.com. PEI looks forward to your continued participation in new programsof continuing education as they become available.66© 2007 Professional Evaluation, Inc.

Hooray! Your file is uploaded and ready to be published.

Saved successfully!

Ooh no, something went wrong!